Алгебра 20 В конец треда | Веб
Если математика - царица наук, то алгебра - венец самой математики. Этот тред посвящён ей! ссым в нём на тех, кто занимается анализом и не знает теорему де Рама

Тред используется для любых вопросов, связанных с современной алгеброй и её ответвлениями (но не ограничивается оными).

Для начала предлагаю следующую задачу на пикрелейтид.
Это самоконтрольный тест. Не можешь её решить - не владеешь алгебраической геометрией.

Заниматься алгеброй — значит, по существу, вычислять, т. е. выполнять над элементами некоторого множества "алгебраические операции".
Несомненно, именно возможность этих последовательных операций, при которых форма вычислений оставалась одной и той же, но природа математических объектов, над которыми производились вычисления, существенно менялась, позволила постепенно выявить руководящий принцип современной математики: математические объекты сами по себе не столь существенны — важны их отношения
.

Архивач
2 36202
>>36117
Мне для теории групп в общем-то, некоммутативные нада.
jvzA1wtjq3U.jpg131 Кб, 960x720
3 36283
Как решить?
4 37398
>>36283
Каким свойством должен обладать многочлен чтобы делиться на x_i-x_j k_i \times k_j раз?
5 38251
Сап, матемач. Проебал по болезни много важных лекций по линейной алгебре. Посоветуйте годный ресурс, где можно почитать про алгебраические структуры (поля, кольца, группы, операции).
6 38258
>>38251
Винберг.
7 38918
Помогите разобраться с тензорным произведением, а конкретней, с разложимыми элементами z=xy ( - это тензорное произведение). Почему они не составляют всего произведения пространств (не в одномерных случаях)? Ещё утверждается, что ранг матрицы координат элемента тенз.произведения в случае разложимых элементов =<1 (собсна, из второго следует первое).
Но при этом ведь любой элемент представляется в виде суммы по i, j: z(ij)ei*fj, где ei и fj - базисыные вектора соответственных пространств. Разве это не одно и то же? Или тут суть именно в том, что у разложимых элементов нет коэффициенты z(ij)?

ток не бейте
8 38919
>>38918
Двач съел звездочку, которая тензорное произведение обозначала, но думаю, и так понятно.
9 38939
>>38918

>z=xy ( - это тензорное произведение). Почему они не составляют всего произведения пространств (не в одномерных случаях)?



Потому что всё пространство это линейные комбинации тензоров такого вида, при этом сами друг с другом они необязательно линейно зависимы (если пространство не одномерное)
10 38952
>>38939
Можешь пример привести? Я именно на фундаментальном уровне недопонимаю это утверждение. И почему для одномерного случая всё нормально.
Почему мы этот коэффициент не можем просто вынести из какого-то векторного пространства в виде xj=СУММА по i (zij(ei)), тензорное произведение же p-линейное. И представить каждый z как сумму по j (xj*ej). Или это уже не то же самое, что разложимый элемент?
11 38960
>>38952
я плохо понимаю твои обозначения, ты можешь использовать tex команды? учи tex, иначе невозможно

>Можешь пример привести? Я именно на фундаментальном уровне недопонимаю это утверждение. И почему для одномерного случая всё нормально.



возьмём $\mathbb{R}^2 \otimes \mathbb{R}^2$ и увидим, что это $\mathbb{R}^4$. В самом деле, базис образуют следующие элементы

$(0,1) \otimes (0,1)$
$(1,0) \otimes (0,1)$
$(0,1) \otimes (1,0)$
$(1,0) \otimes (1,0)$

Я утверждаю, что тензор $(1,0) \otimes (1,0) + (0,1) \otimes (0,1)$ не разложимый. В самом деле, рассмотрим билинейное отображение $b$ из $\mathbb{R}^2 \times \mathbb{R}^2$ в матрицы $2\times 2$, которое парам $(a_1,a_2)$ и $(b_1,b_2)$ сопоставляет матрицу $(a_i b_j)$, $i,j =1,2$. Тогда, если рассматривать пары $(a_1,a_2)$ и $(b_1,b_2)$ как один тензор $(a_1,a_2) \otimes (b_1,b_2)$ здесь мы используем универсальное свойство, то легко видеть, что на всяком разложим тензоре $v \otimes w$ матрица $b(v,w)$ имеет нулевой определитель, в то время как на тензоре $(1,0) \otimes (1,0) + (0,1) \otimes (0,1)$ получается единичная матрица
12 38965
>>38960
Так, техать я умею, но хром его тут не отображает. Расширение какое поставить? Или просто скопировать куда-то?
А пояснении разберусь, традиционное нихуя спасибо от меня.
13 38967
>>38965

>но хром его тут не отображает



не надо отображать, все всё понимают все свои
14 38974
>>38967
А неофитов не принимаете, да?
15 38988
>>38974
пусть учатся, что не так?
16 38989
>>38960
Кажись понял, благодарю
17 38990
Тензорное произведение Z-модулей Q/Z на Q же равно нулю?
18 38993
>>38990
да, разумеется.

возьмём простой (разложимый тензор) вида $a \otimes b$, где $a = x/y + \mathbb{Z}$, $b \in \mathbb{Q}$. тогда имеем
$$
(x/y + \mathbb{Z}) \otimes b =
(x/y + \mathbb{Z}) \otimes y b/y =
y(x/y + \mathbb{Z}) \otimes b/y =
(x + \mathbb{Z}) \otimes b/y =
0 \otimes b/y = 0.
$$

Мы воспользовались линейностью по $\mathbb{Z}$.

Коль скоро любой тензор есть линейная комбинация простых тензоров, а все простые у нас равны нулю, значит, и все возможные тензоры равны нулю. Так, искомое произведение состоит из одного нуля.
19 38997
Парни, высрал решение задачи, зачекайте, я полную хуйню написал или все норм?
20 38998
>>38997
читать очень трудно, но думаю, изоморфизм указан правильно. надо проверять, что всё корректно (не зависит от выбора представителей). линейность более-менее очевидна
21 39009
>>38997
Помимо формул, русским языком и аккуратным почерком распиши нормально то, что ты написал, это неуважение такую хуйню притаскивать.
Подлежащие, сказуемое, дополнение.
Необучаемые, блядь.
22 39039
>>38997
Это вообще неверно, так как инъективность гомоморфизма i : M' -> M не обязана сохраняться при переходе к гомоморфизму M' \otimes N -> M \otimes N. То есть, модуль M' \otimes N не обязан быть подмодулем M \otimes N. Пример - любая конечная абелева группа тензорно на Q тривиальна.
Скорее тут имеется ввиду, что M/M' \otimes N изоморфно (M \otimes N) / im (i \otimes 1_N), то есть коядру гомоморфизма i \otimes 1_N. А это уже тривиально следует из того, что тензорные произведения сохраняют точные последовательности "справа" (но не обязательно слева, так как нет сохранения инъективности).
23 39529
Как найти полярное разложение вырожденного оператора A? У меня вышло, что AA^T - диагональный и вырожденный, т.е. S=sqrt(AA^T) тоже вырожденный и найти ортогональную матрицу по принципу U=S^(-1)A уже нельзя.
24 39542
>>39529
ох, как я всё это позабыл

>У меня вышло, что AA^T



поясни, что ты понимаешь под "диагональный линейный оператор"?
25 39557
>>39529

>матрицу


Что это?
26 39559
>>39542
Ну, типа он уже диагонализирован и на диагонали стоят собственные значения.
27 39560
>>39557
Не понял вопроса
28 39578
>>39559
тогда это правильно, его можно привести к такому виду, потому что симметричный. он вырожденный, если A вырожденный
29 39877
Посоветуйте тексты по когомологиям пучков.
30 39888
>>39877
любой текст математический подойдёт, ведь вся математика это частные случаи пучков
31 39894
>>39888
Это понятно, но любой математический текст не обязан обсуждать школьную математику, т.е. когомологии пучков.
32 39905
Мне тут накидали буксы. Алгебру до этого не изучал, кроме обычной школы. Какой букс лучше выбрать для первоначального изучения для студентов-математиков?
1. https://math.berkeley.edu/~apaulin/AbstractAlgebra.pdf
2.http://ebooks.bharathuniv.ac.in/gdlc1/gdlc1/Mathematics Books/Introduction to Modern Algebra - David Joyce.pdf
3. Algebra: Chapter 0. Paolo Aluffi.
4. http://users.metu.edu.tr/sozkap/461-2010/book.pdf
5.https://homepages.warwick.ac.uk/~maseap/teaching/aa/aanotes.pdf
6.https://math.dartmouth.edu/archive/m31x13/public_html/Notes on Abstract Algebra 2013.pdf
7. http://users.metu.edu.tr/serge/courses/116-2015/Textbook116.pdf
33 39911
>>39529
Но ведь на ортогональном дополнении к своему ядру S невырождена, правда? Найди кусок U там, а остаток добей как попало до ортогональности, всё равно его сожрёт S.
34 40682
Дайте гайд по Ротману.
35 40749
>>40682
Твой вопрос непонятен.
36 40872
Матаны, поясните меня, нуба, по хардкору за решетки.
37 41703
В процессе изучения операторов столкнулся впервые с понятием топологии и читать книги про это как-то не хочется, ибо там тема эта только в четверти параграфа есть. Пытался по определениям быстро разобраться, но там их что-то слишком много и они про разные вещи, как я понял.
1. Что вообще такое топология (не в смысле раздела математики)? Система подмножеств, удовлетворяющая тем аксиомам про пересечение и объединение?
2. Что имеется ввиду, когда говорят, что на конкретном множестве можно задать различные топологии? Имеются в виду, что задаются различные базы топологий?
3. Когда говорят, что различными нормами на пространстве определяются различные топологии, то что тут имеют в виду? Норма задаёт базу топологии или это разные вещи? Имеется в виду, что норма задаёт метрику, а метрика задаёт топологию? Всё опять упирается в открытые шары (всегда ли можно открытые шары, если они определены, заменить открытыми кубами или нет)?
38 41705
>>41703
1. Да.
2. Что для множества можно предложить разные системы его подмножеств, удовлетворяющие аксиомам топологии. Два классических примера - дискретная и антидискретная топологии. Если во множестве больше одной точки, эти топологии будут разными.
3. Норма задаёт метрику, метрика задаёт топологию (базой которой являются открытые шары).
Вообще говоря, шары можно ввести в любом метрическом пространстве, а кубы специфичны для R^n.
39 41707
>>41703

>читать книги про это как-то не хочется, ибо там тема эта только в четверти параграфа есть


Зря. Топология, как и линейная алгебра, основа математики.
40 41709
>>41705
Спасибо.
>>41707
Всему своё время, думаю, потом эту тему всё равно придётся подробно разбирать.
41 41852
Автоморфизмы - взаимо-однозначное отображение, так? То есть это различные подстановки? Тогда почему на вопрос сколько автоморфизмов на подстановках от n элементов ответ не n! ?
42 41854
>>41852
Автоморфизмы структуры - это обратимые гомоморфизмы структуры в себя. Например, если G - группа, её автоморфизмы - не все возможные биекции G->G, а только такие биекции f, что f(ab) = f(a)f(b). Количество автоморфизмов зависит от рассматриваемой структуры - чем больше отношений должно сохраняться (т.е. чем богаче сигнатура структуры), тем меньше будет автоморфизмов.
43 41894
>>2623
Что скажешь про Курроша?
44 41956
>>35672

>Дьедонне


Двачую адеквата
45 41980
>>41894
на уровне книги Дуракова
gomo.png4 Кб, 377x42
46 42031
Надо сделать для перестановок.
Для а) думаю, что это: 1) отображение, переводящее каждую подстановку в ту же самую 2) отображение, переводящее каждую подстановку в следующую за ней.
Для б) такое же как и в 1 для а) и ещё отображение, переводящее (21) -> (123). Как найти другие? Если перемножать (21) последовательно на подстановки из S_3 получатся подстановки: 3,4,1,2,6,5. Из этой информации можно что-то получить?
Как делать для следующих пунктов?
47 42032
>>42031
Похоже не учёл ещё подстановки из двух элементов в S_3. (12), (21), (13), (31), (23) и (32) принадлежат S_3?
48 42045
>>42031

>и ещё отображение, переводящее (21) -> (123)


Ты переводишь перестановку порядка 2 в перестановку порядка 3? No way.

>Если перемножать (21) последовательно на подстановки из S_3 получатся подстановки: 3,4,1,2,6,5.


>3,4,1,2,6,5


Чё?
И зачем перемножать. Сопрягать надо, раз так.

>Как делать для следующих пунктов?


Гомоморфизмы из S_2 — это элементы порядка 2, а это (мы ведь знаем что такое цикленный тип?) перестановки цикленного типа (2, 2, ..., 2) (сколько-то двоек), то есть произведения дизъюнктных транспозиций.
В S_3 подгруппа индекса 2 (= порядка 3) только одна (3-цикл).
>>42032
(12) и (21) — это одно и то же.
49 42074
>>42045

>Сопрягать


Почему? Насколько понял гомомофизм - отображение φ : S n → S m, если φ (ab) = φ (a)φ (b) при любых a, b ∈ S n .

>перестановки цикленного типа


Тогда образ гомоморфизма: (21), (31), (32)?
50 42080
>>42074

>Почему? Насколько понял гомомофизм - отображение φ : S n → S m, если φ (ab) = φ (a)φ (b) при любых a, b ∈ S n .


Сопряжение фиксированным элементом — автоморфизм, композиция гомоморфизма с автоморфизмом снова даёт гомоморфизм.
[Неважно, забей.]

>Тогда образ гомоморфизма: (21), (31), (32)?


Если (надеюсь) я правильно тебя понял (есть три гомоморфизма, каждый из них определяется тем, что переводит нетривиальный элемент S_2 в какой-то из трёх этих элементов S_3) — да. Но образ кажого из этих гомоморфизмов имеет вид {1, (ij)}, лучше всё-таки употреблять слова по назначению.
51 42094
>>39905
Начать с Винберга, потом переходить на Aluffi
52 42162
Как найти сумму и пересечение пространства многочленов, делящихся на фиксированные многочлены p 1 , p 2 ∈ R [x]?
53 42168
>>42162
Что значит «найти»?
сумма: многочлен p_1 + многочлен p_2
пересечение: многочлен p_1 p_2
(многочлены вида...)
54 42169
>>42168
Это произведения, форматирование превратило в курсив. Похоже, что надо × использовать, он и выглядит лучше. Или точку: a⋅b.
lin.png10 Кб, 471x66
55 42197
Вопрос: являются ли линейными следующие отображения A: L_1→ L_2.
По е):
Верно, что нет. Потому что не A(x+y) не равен A(x)+A(y).
Как ж) проверить?
56 42207
57 42214
>>42197
Если я правильно понял, то в ж) каждой сходящейся последовательности ставится в соответствие её предел. Т.к. lim(a+b)=lim(a)+lim(b) доказывается в любой книжке по анализу, то отображение линейно.
58 42215
>>42214
И под е) линейное отображение
У тебя отображение полиномов в полиномы. Нужно не (x+y) в полиномы подставлять, а брать сумму полиномов (p+r). Отображение линейно из-за дистрибутивности: q(p+r)=qp+qr.
59 42244
Не уверен, что вопрос стоит адресовать этому треду, но всё-таки: имеют ли группы Ли приминение в физике и если да, то в каких разделах?
60 42247
>>42244
В теории квантового поля, в некоторых изложениях пол-курса затирают про эти группы Ли. Преобразования Лоренца/Пуанкаре образуют группу Ли, а их (унитарные) представления соответствуют частицам с разными спинами. (и это чертовски занятный факт имхо)

Вот тебе еще какая-то мутная теоремка из квантовой механики
https://en.wikipedia.org/wiki/Wigner–Eckart_theorem
61 42267
Векторы множества всех геометрических прогрессий с первым членом, равным 1, являются линейно независимыми, так как можно выбрать взаимно-простые знаменатели?
62 42284
>>42244
Да, имеют. Вращения — это группа Ли. Сдвиги пространства — это группа Ли. Уже по этим примерам видно, насколько это (простое и) важное понятие. Классификация элементарных частиц использует группы Ли, например, кварки — это просто какое-то представление какой-то из SU (комплексный аналог вращений), а то, что всё состоит из кварков — это то, что все представления получаются из этого «простого» представления с помощью произведений (тензорных). Подробнее, например, в книге И.Р. Шафаревич «Основные понятия алгебры» (там есть пункт про это).
Представление — сопоставление элементам группы линейных преобразований, вроде представления перестановок трёх букв как симметрий равностороннего треугольника; тензорное произведение — полный аналог перехода от однородных многочленов степени 1, которые можно отождествить с векторами, к многочленам произвольной степени, только абстрактно аксиоматически описанный для абстрактно аксиоматически заданных линейных пространств. Это если кто-то не знает, вдруг.
63 42313
>>42284
>>42247
Спасибо за ответы
64 42319
Сколько различных базисов существует в ( Zp)^n? p^n?
65 42321
>>42319
Если под Zp имеется в виду Z/pZ, то количество базисов равно Π(p^n - p^i), i=0,...,n. Но это упорядоченных базисов.
66 42329
Как доказать, что rk(BA) + rk(AC) ⩽ rk(A) + rk(BAC)?
67 42345
Пусть (e1,...,en ) — базис L; (g1,...,g n) — дважды двойственный
ему базис L^tt, A ∈ Hom(L, L^tt ) — такое отображение, что A(e_i) = g_i. Зависит ли A от выбора базиса (e_i)?
Верно, что дважды двойственный базис L^tt это базис L? Значит, A от выбора базиса не зависит.
68 42367
>>42345
Забей на базисы.

Смотри. Есть линейное пространство L. Пространство функционалов f(v) на нём есть двойственное к нему L'.

Выберем какой-нибудь определенный v из L, и подставим его во все функционалы в L'. Не совсем строго можно теперь считать, что это не функционал ставит число вектору v, а вектор v ставит функционалам числа, тем самым такое отображение есть функционал. Тем самым вектора в L это функционалы для пространства L'.

Чтобы записать это строго нужно построить функцию, которая каждому вектору v ставит соответствующий ей функционал на L'. h: v = gv(f) где gv(f)=f(v). Так как все двойственные пространства имеют одинаковую размерность, то нужно всего лишь доказать, что h мономорфизм, оттого h изоморфизм.
69 42389
>>20 (OP)

>математические объекты сами по себе не столь существенны — важны их отношения.


Что это значит?
WhzJLTo-NDg.jpg134 Кб, 1113x771
70 42441
В процессе чтения Винберга возник вопрос.
G={e,a,b,c} — абелева группа с таблицей умножения пикрил.
Не могу понять, почему перестановка элементов a, b, c является автоморфизмом G с указанной операцией (той, которая таблице задаётся, да?).
Почему в перестановке участвуют только a, b, c, что делать с e?
Как строить-то этот автоморфизм?
71 42449
>>42441
При автоморфизме e в любом случае переходит в e, так что ясно, что с ней делать.
Таблица умножения по сути говорит, что каждый элемент обратен сам себе и что произведение двух разных неединичных элементов равно третьему неединичному элементу, она симметрична относительно всех перестановок неединичных элементов. Поэтому эти перестановки и определяют автоморфизмы.
72 42469
>>42449
Автоморфизм строится аналогично тому, который встречается при доказательстве теоремы Кэли?
f(g_j): g_i → g_j⋅g_i, где f суть есть перестановка.
73 42470
>>42469
Автоморфизм, естественно, не f (перестановка, описанная выше), а другое отображение, к примеру, L: g_j → f(g_j).
Это всё ещё вопрос, а не удтверждение. Верно?
74 42499
>>42469
>>42470
Был неправ, проспался и исправился.
75 42570
Верно, что подгруппа (2Z)⊂Z нормальна? Потому что нечёт+нечёт=чёт, а значит из (2Z) не выпадает при сложении с элементом из (2Z).
76 42575
>>42570
Любая подгруппа абелевой группы нормальна.
77 42592
Как определяется понятие движение без понятия расстояния?
79 42602
>>42592
Движение -- это конгруэтность.

Две фигуры подвижны(одна переходит в другую), если равны параметры фигуры.
80 42603
>>42592
Очевидно, что движение без расстояния, значит форму без метрики.
Значит единичный куб представляет ту же фигуру, что и стократный куб.
81 42851
Как доказать что симметрическая разность и два множества изоморфны z/2z?
82 43405
Сап, матач. Дрочу на алгебру, абстракции всякие(пока только в пределе курса в унике). Но хочу применять это, в частности, в машинке. Какие подводные?
83 43423
>>43405
Подводные в том, что в машинке это не применяется.
84 43424
>>43405
Вкатывайся в криптографию.
85 43436
>>43424
я купил биткойны в прошлом декабре
86 43754
cos(2*pi/5) можно выразить через рациональные числа?
87 43760
>>43754
x = 2pi/5
Подсказка: cos(2x) = cos(3x), следовательно можно выразить cosx через формулы двойного и тройноог угла и получится уравнение.
88 43851
Известные корни кубического уравнения x^3+px+q=0.
Сделал несколько преобразований получил:
2q^3(1/x1^3+1/x2^3+1/x3^3)+6q^2-q^2(x1^2/x3^2+x1^2/x2^2+...)-2q^2(x1/x2+x1/x3+...)+2q(x1^3+x2^3+x3^3). Дальше увяз.
Правильно понимаю, что (x_1-x_2)^2(x_1-x_3)^2(x_2-x_3)^2 - дискриминант кубического уравнения?
89 43852
>>43851
Преобразования делал из того что написано внизу. (x_1-x_2)^2(x_1-x_3)^2(x_2-x_3)^2
90 43854
>>43851
Правильно. В чем, собственно, вопрос? Что ты хочешь получить?
91 43860
>>43854
Вопрос в том как грамотно от (x_1-x_2)^2(x_1-x_3)^2(x_2-x_3)^2 перейти к -27q^2-4p^3. Под спойлером у меня записано на чём я остановился.
Раз это и правда дискриминант, тогда ещё один вопрос. Почему именно это число связано с корнями многочлена? Положим, я не знаю, что (x_1-x_2)^2(x_1-x_3)^2(x_2-x_3)^2 - дискриминант кубического уравнения. Тогда как к нему прийти от редуцированного уравнения x^3+px+q?
92 43874
Как сейчас в мире обстоят дела с теорий магм?
Какие там есть интересные задачи?
93 44007
>>42851
Руками, по определению, непосредственно.
94 44017
Решил повторить всю математику с первого класса. Мне 33 годика
95 44063
>>44017
Зачем с первого? Начни с категорий для рабочего.
96 44075
>>44063
Заводобыдло не нужно
с.jpg56 Кб, 655x1024
97 44094
>>44075
это было так тонко, что даже толсто. лол
Аноним 98 44167
>>20 (OP)
Знает кто доказательство теоремы о определителе треугольной матрицы

>Определитель матрицы треугольного вида равен произведению элементов, стоящих на главной диагонали.


Как доказать?
99 44168
>>44167
Просто посчитать по формуле.
Аноним 100 44183
>>44168
Как?
101 44186
>>44183
Берёшь и расписываешься по столбцу или строчке, замечая, что если дополнительный минор содержить нулевой столбец, то он равен 0.
102 44199
>>2247

>Дифференцирование это гомоморфизм кольца.


Чё за хуйню я только что прочитал
103 44200
>>44199
свою безграмотность, очевидно же
https://en.wikipedia.org/wiki/Derivation_(differential_algebra)
104 44202
>>44200
ВО 1ых, найди в той статье слово "гомоморфизм", ты его не найдешь знаешь почему? Потому что дифференциал произведения не равен произведению дифференциалов.
Во-вторых, научи так же шутить "что я за хуйню прочитал?" - "свою безграмотность, очевидно же"! просто АХУЕТЬ! Как ты только блять додумался до такого, ебучий придурок сука!
105 44208
>>44202
все, обтекаю
Аноним 106 44211
>>44186
спасибо
107 44220
>>44202

>о дифференциал произведения не равен произведению дифференциалов.


но сумме то равен
108 44228
>>44220
И поэтому это гомоморфизм кольца? Или к чему ты это пизданул?
109 44233
Аноны, есть какая нибудь методичка по решению базовых задач по алгебре?
110 44899
Посоны, в шараге заставляют учить ТЧ по Бухштабу и Виноградову. Посоветуйте норм учебник по ТЧ, но чтобы там были все эти знаменитые китайские теоремы и ебля с вычетами.
111 45001
>>44899
Манин.
112 45137
Как показать, что единственный тензор нечётного ранга, инвариантный относительно вращений и симметричный относительно перестановки любой пары индексов, это нулевой тензор?
И как вывести общую формулу для тензора чётного ранга, инвариантного относительно вращений и симметричного относительно перестановки любой пары индексов? Там сумма произведений символов Кронекера должна быть.
113 45138
>>45137
Во втором полагаю, что надо произведенией n/2 символов кронекера симметрировать, где n - ранг тензора + ещё учесть числовой множитель как-то.
Выражение дробей 114 45603
Двачик как находить выражения дробей? Научите
115 45708
Металлофизик на связи. Поясните, для приложений в физике (группы симметрии, алгебраическая топология и т.д.) есть ли необходимость проходить полноценный курс по высшей алгебре (типа Rotman, Lang), или достаточно ограничится книжками типа "Group Theory in a Nutshell for Physicists"? Если ориентироваться на перспективу в той же физике и различных разделах математики?
Какие ориентированные на упражнения книги можете посоветовать (желательно с решениями или ответами для самопроверки, можно на английском)?
116 45746
>>45708
В металлофизике нужна алгебраическая топология?
117 45809
>>45746
Да, атомы в кристаллическую решетку как без нее загонять?
118 45814
>>45809
Бля, серьезно же спросил.
119 45817
>>45814
да он сам не знает, потому и спрашивает у нас
только мы сами не физики и ответить не можем. странный он, какой ответ он ждал-то?
120 45857
>>45814
Я вообще мимо прохоил и решил пошутить.

Наверное топология просто вылезла в каком то неожиданном месте в металлофизике, раз её реквестируют.
121 45904
Какие 8-элементные группы разлагаются в нетривиальные полупрямые произведения?
122 45907
>>45746
>>45857
Нобелевка по физике за 2016
123 45940
>>45857
В общем-то в condensed matter всё уже пропитывается всякими гомотопиями и категориями, см. например, https://arxiv.org/pdf/1810.12965.pdf
124 46068
Z/mZxZ/nZ изоморфно кольцу Z/mnZ только при взаимно простых m и n?
125 46450
Подскажите где взять материал про лемму Шура без обращения к теории представлений,а использовать модули.
126 46458
>>46450
модуль это и есть представление.
127 46460
>>46068
пусть m и n имеют общий делитель a...
128 46464
Есть ли какой то лучший способ нахождения перестановочных подстановок, чем тупой их перебор?
129 46487
>>46464
Конечно, это очень легко. Централизатор перестановки состоит из перестановок, которые переставляют&прокручивают её циклы.
130 46500
>>46487
Я должен знать что такое централизатор и как это применять учась на первом курсе?
131 46514
>>20 (OP)
Скиньте плез билеты по алгебре первый семестр для матфака, условной вышки или мгу. Если можно то все 3 семестра. Буду благодарен, если скинете также по дискретке/графам/алгоритмам и прочей информатике.
132 46518
>>46500
Ну если ты знаешь слово "группа", то конечно.
134 46524
бамп
ОСНОВЫ ЛИНАЛА 135 46557
Не вкуриваю в собственные вектора/значения. Допустим, у меня есть диагональная матрица:
|2 0|
|0 2|
Существуют четыре прямые, вдоль которых она растягивает вектора, не меняя их направления: по горизонтали/вертикали с коэффициентом 2; по диагоналям с коэффициентом (8)^(1/2).
Почему же утверждают, что у неё может быть не больше 2 неколлинераных собственных вектора?
136 46577
>>46557

>Существуют четыре прямые, вдоль которых она растягивает вектора, не меняя их направления: по горизонтали/вертикали с коэффициентом 2; по диагоналям с коэффициентом (8)^(1/2).


Уверен, что только 4?

>Почему же утверждают, что у неё может быть не больше 2 неколлинераных собственных вектора?


Кто утверждает? Сам же опроверг это утверждение.
137 46579
>>46577

>Уверен, что только 4?


Вроде, да. Вектора, направленные вдоль других прямых, изменят угол наклона.

>Кто утверждает? Сам же опроверг это утверждение.


"Оператор A (матрица A) имеет не более n различных собственных значений (в n-мерном линейном пространстве)" - например, тут:
http://twt.mpei.ac.ru/math/LARB/Linoper/LA_04050000.html
138 46580
>>46557
Твой оператор тупо умножает вектор на два, поэтому для него ВСЕ ненулевые векторы собственные. Смысл в том, что нельзя взять больше двух векторов так, чтобы они были линейно независимы . Даже если взять только те, что ты перечислил, то для них это выполняется.
139 46584
>>46557
Когда ты ищещь собственные векторы, ты, вообще говоря, ищешь собственные подпространства, т.е. базисы подпространств, векторы которых растягиваются в число раз, соответсвующее числу соответствующего собственного значения.
У тебя любой базис двумерного простраснтва состоит из двух векторов. Так что очевидно, что совокупность базисных векторов собственных подпространств не может превзойти совокупность базисных векторов всего пространства.
140 46585
>>46577

>собственных значений


а не собственных векторов

>Вроде, да. Вектора, направленные вдоль других прямых, изменят угол наклона.


Никакие углы не меняются. Ты знаешь, как перемножаются матрицы?
141 47629
Периодическая часть абелевой группы обязательно является её прямым слагаемым?
142 47637
>>47629
Вряд ли, недаром же у нас теорема классификации есть только для конечно-порождённых
143 47664
>>46514
Зайди на сайт.
144 48921
Вопрос простой есть две переменные a и b по аргументам алгебры переменные это множитель при той или иной степени неизвестного получается и в А и В могу ли я вместить 5 то получается квадрат разности и разность квадратов лжет извините но просто нигде не нашел про это информацию
145 49001
ЧТО ЭТО КАК ЭТО РЕШИТЬ
146 49002
>>49001
Никак, это не уравнение.
147 49003
>>49002
Спасибо, добрый анон.
148 49254
Артин или Лэнг?
149 49255
или Винберг, лол
150 49369
>>46519
А есть подобное от Чебышевки?
151 49851
Почитал описания кольца на Википедии, пришел к выводу, что это тоже самое, что и линейное пространство, нет?
152 49852
>>49851
Не пиздите меня пожалуйста, я знаю, что я тупой, но я только начал изучать вышмат, параллельно готовясь к ЕГЭ это не так просто
153 49858
>>49852
Векторное пространство строится над полем (частным случаем кольца). Разъясни для себя хорошенько этот момент.
154 49860
>>49858
Какой есть учебник, написанный понятным языком? А то как не начну читать ваших Винбергов и прочих, так удушить себя хочется, какая же сложносформулированная залупа там
155 49861
>>49860
Хз, что может быть проще Винберга или Кострыкина (вводные курсы ведь), в любом случае надо учиться воспринимать такую подачу. Там вроде бы и примеры приводятся + гугол есть.
156 49864
>>49851
Нет. В линейном пространстве нет умножения элементов, вместо этого - умножение на элементы основного поля.
157 49868
Читаю ленга, можем вникать вместе tg svetocopyclassic
158 49920
>>49860
Алуффи
159 50001
>>49860
M. Artin
image.png15 Кб, 595x60
160 50696
двачик, никак не могу понять ход решения задачи. Ответ a >= 11/5
sage 161 50711
15537460018173.webm4 Мб, webm,
1280x720, 0:20
162 52280
Объясните нубу. Чем занимается алгебра? То есть, я могу взять придумать какую-нибудь систему любые аксиомы и любые операции, но по итогу я в ней найду те же свойства, особенности, всякие структуры, которые есть в других системах? И вот эти свойства и особенности которые есть везде и изучает алгебра? Я верно понял?
163 52282
>>52280
Ты можешь взять и придумать множество (например расширение Q каким-нибудь иррациональным числом; ещё можешь наложить дополнительные условия в виде тождеств которым должны удовлетворять любые два элемента твоего множества), и можешь придумать себе операцию, простой пример это "сложение с нулем", или например последовательную комбинацию сложения, умножения и возведения в степень, или ещё что-то.
И потом проверить, какие аксиомы выполняются, является ли то что ты придумал кольцом или группой. Здесь в принципе два варианта. Либо это уже известный пример и ты изобрел вещественные или комплексные числа; либо оно вообще никакой разумной структуры не образует и не является в алгебраическом смысле ничем.

Это в принципе мало интересное упражнение. Интереснее находить известные структуры там, где их, казалось бы, нет. Например то что кобордизмы можно умножать и складывать и выполняются все аксиомы кольца. Или там нетривиальные операции в когомологиях, квадрат Стинрода, произведение Масси. Внешнее произведение поливекторов, или дифф. форм, или скобка Пуассона. В принципе это фундаментальной важности было открытие, что как алгебраическая структура, алгебра дифференциальных форм изоморфна внешней алгебре. По сути ты показываешь, что что-то до сих пор плохо и недостаточно понятое оказывается можно описать в известных и привычных терминах. В целом, исторически, нахождение соответствующей структуры позволяло а) исправить ошибочные утверждения б) перейти к более общей ситуации (например понятие модуля сначала возникло в работах Дедекинда, и только потом выяснилось что если векторное пространство определить над кольцом, то получится идентичная конструкция), в) найти двойственные объекты и операции (ковариатные и контравариантные векторы, полупрямое произведение и т.д.)
164 52284
>>52280
Это очень условное деление. Например, топологическое пространство - это тоже множество с заданной некоторыми аксиомами структурой, но само по себе оно алгебраическим объектом не считается.
Если очень нужно простое и ёмкое определение, то можно, к примеру, с натяжкой определить алгебру как науку об ассоциативных бинарных операциях на множествах. Что-то мы при таком определении упустим, но на то это и условное деление.
165 52289
>>52284
>>52282
Я имел в виду не только числа, а вообще любые объекты в том числе и топологические. Значит я не правильно понял, что алгебра изучает структуры, которые существуют вообще в любой структурированной системе? Раз есть системы неалгебраические. А можно ли любую систему сделать алгебраической не включая в нее ограничения и операции, которых там не было?
166 52290
>>52284

>об ассоциативных


а что такое ассоциативных?
Ещё я хотел спросить. Саватеев на своей первой лекции сказал что в математике не бывает правильных и неправильных дробей. И что типа дробь 48/16 это типа нормальная крутая дробь. Но какого хуя то? Как можно думать таким понятием?
167 52291
>>52289
Операцию предельного перехода, понятие непрерывности и всё такое, невозможно описать алгеброй. Для этого придумали топологию. 99% математики это переплетение алгебры и топологии. Например, функциональный анализ. Берем алгебраический объект (векторное пространство), добавляем топологическую структуру (пополняем по норме).
Или гомологическая алгебра. И т.д.

>ассоциативных


Ну тех где выполняется аксиома ассоциативности, очевидно, a (b c) = (a b) c.

>правильных и неправильных дробей


Про рациональное число лучше всего думать как про класс эквивалентности, 5/15 и 1/3 это не разные числа, просто разный способ записи одного и того же числа. То же можно сказать про 0.99999… и 1.
168 52292
>>52291
а числа 0,(9) не существует. это дыра на числовой прямой. Это я ещё у одного блогера узнал
169 52293
>>52292
Какая может быть дыра, если $R$ всюду плотно?
170 52294
>>52291
Тогда получается можно в любой системе с любыми объектами найти одинаковые принципы. А это либо топологические либо алгебраические принципы? То есть алгебра и топлогия занимается этими универсальными принципами?
171 52295
>>52294

>можно в любой системе с любыми объектами найти одинаковые принципы


А можно и не найти. Но найти это всегда хорошо.

>либо топологические либо алгебраические принципы


Теоретико-категорные.
172 52296
>>52295
Начинаю путаться. Короче, если я хочу универсальных принципов, то мне надо теорию категорий изучать. А алгебра и топлогия это частные случаи.
173 52297
>>52293
Ты хотел сказать "полно", а не "всюду плотно".

>>52296
Да, тебе надо изучать теорию категорий. Ещё можно математическую логику, это в принципе тоже об универсальных принципах, но с другого бока.
Алгебру и топологию придётся изучать в любом случае, без базовых знаний в этих областях ты не вдуплишь большую часть примеров, которые будут тебе нужны для тех же категорий.
174 52298
>>52290
Савватеев имеет ввиду, что в большинстве случаев к неправильным дробям в ответе доёбываются по беспределу. С корнями в знаменателе такая же история.
175 52304
>>52294
Я алгебру воспринимаю как прерывную математику, а топологию как непрерывную. Алгебра кубик Рубика, топология пластилин, чета такое.
176 52313
>>52304
А любое упоминание о группах Ли или топологических векторных пространствах причиняет тебе мучительную боль? Или, вероятнеё, ты с такими понятиями не сталкивался
177 52314
>>52313
Мне кажется это уже алгебраическая топология/геометрическая алгебра или что-то такое, комбомикс. Полностью разделить невозможно ведь.
178 52315
>>52314

>геометрическая алгебра


Топологическая.
фикс
179 52318
>>52314
Ну хорошо, а вот комбинаторика не связана с непрерывностью, она тоже к алгебре относится?
180 52319
Смотрите, вот Арнольд предлагает определять группу не через операцию и её аксиомы, а как некоторое множество преобразований. В принципе, имеет право, ведь любая группа вкладывается в группу биекций на некотором множестве.
Но можно ли подобным образом определить, например, полугруппу или модуль? Полугруппа, возможно, вкладывавется в полугруппу всех отображений множества в себя, но я не видел доказательств этого факта. А с модулем вообще сложно. Например, любое ли кольцо вкладывается в кольцо эндоморфизмов абелевой группы?
181 52321
>>52319
Половина языкочесательных выступлений Арнольда - о преподавании. У тебя есть опыт преподавания? У многих тут есть, как ни странно, в том числе и у меня.

Если тебе нужно рассказывать про полугруппы или модули, то студенты, вероятно, уже обладают достаточной математической культурой для понимания таких понятий. Тем более, полугруппа - плохой пример, это просто множество с ассоциативной операцией, там не нужно (было бы) танцевать, объясняя это школьникам.

И да, Арнольд так определял группу в своих лекциях школьникам о теории Галуа, и оно действительно интуитивнее и полностью обосновано в этом случае.

Ещё меня веселит тот факт, что вербито-адепты ненавидят/презирают Арнольда за такие вот определения, хотя они по духу куда ближе к категорному языку, чем традиционные.
182 52322
>>52321
Любой критикующий арнольда это вербитоадепт?

>действительно интуитивнее и полностью обосновано в этом случае


Оно интуитивно, конечно, и не бесполезно как иллюстрация, особенно если речь о конкретных примерах, например группе симметрий тетраэдра и её подгруппах. См. "Что такое математика", он отсюда и приходит к понятию короткой точной последовательности. Более того, ещё было бы небесполезно сказать пару слов о подстановках, и показать что симметрии и подстановки это одно и то же (можно просто пронумеровать грани), чего Арнольд например не делает, но зато делают другие авторы, например Александров который П.С. в своей брошюре для старшеклассников. При чем у него там есть аксиоматическое определение группы. И теоремы Нётер об изоморфизме он доказывает.

>Арнольд так определял группу в своих лекциях школьникам о теории Галуа


Только дело в том, что это вообще не определение. И тот факт, что ты не понимаешь, что такое определение, полностью объясняет то, что ты не математик, а тупой пиздобол, который ищет вербитодетей на дваче, у себя под кроватью, и в прочих местах.
И критика в адрес Арнольда вообще не на этом основалась, а на ахинее, которую он не только изложил в многочисленных брошюрах, названных кем-то "сборником анекдотов", но еще и прочитывал неоднократно в школьником на летней школе в Дубне и в других местах, и если бы не было пиетета вокруг академической репутации Арнольда, его бы по-хорошему следовало послать нахуй с такими лекциями.
183 52323
>>52322

>пронумеровать грани


Вершины, то есть.
184 52324
>>52321
Полегче, я не залупаюсь на Арнольда, маленький ещё. Мне просто интересно, есть ли аналогичные теоремы для других классов алгебраических объектов.
185 52325
>>52322

>Любой критикующий арнольда это вербитоадепт?


ну ты-то совсем не палишься, у тебя посты как у димки с мишкой один в один
мимо-анон, арнольда не читал
186 52326
>>52318
Нет, да, ладно, плохое разделение.
187 52327
>>52325
Никогда не видел их посты об Арнольде.
мимоВербито-адепт
188 52329
>>52322

>Только дело в том, что это вообще не определение.


хуле нет то? Группа это нечто + тройка морфизмов.
189 52337
>>52322
Сходи тифаретничка ещё наверни, шизик. Ему арнольд только что явно морфизмы не называет, а он продолжает яро защищать убогое дефолтное определение. Теоркат вообще читал дальше введения? Пиздуй в /sci/ нубов гонять, здесь ты никого своими знаниями википедии не поразишь.
190 52339
>>52337

>Пиздуй в свои родные разделы - ну там /b/, /pol/, жж, лурк.


>Пиздуй в /sci/ нубов гонять, здесь ты никого


>Сходи тифаретничка ещё наверни, шизик


Не то что? Порвешься совсем, свинья безмозглая?

>Теоркат вообще читал дальше введения


Ну ты категории по Арнольду учил, я уже понял. Какое у него определение категории, кстати?

>убогое дефолтное определение


Убогое в твоем арнольдистском манямирке разве что, у Маклейна такое же определение группы, как у меня.
191 52365
>>52339
забавно, что ты не понимаешь даже прямого текста, с которым к тебе обращаются, зато пихаешь пафосные выверты типа

>И тот факт, что ты не понимаешь, что такое определение, полностью объясняет то, что ты не математик, а тупой пиздобол



понаблюдал за тобой из соседнего треда
и чего тебя так разрывает любое упоминание арнольда? он до тебя домогался, когда тебе было 9 лет? так об этом надо на фейсбуке писать, вот это был бы наброс
192 52371
>>52339
Представляешь, здесь сидит более одного анона, которые считают, что ты глупенький. Ясное дело, что это вызывает у тебя когнитивный диссонанс и легче всё списать на семёнство, чем на собственное невежество.
193 52380
>>52329
Группа это нечто, да. А ещё группа это точка. А дифференциал это стрелочка.
Если ты хочешь чтобы твои

>глубокомысленные выверты


сравнивали с определениями, которые были у Нётер и Гильберта, потрудись, во всяком случае, выражаться внятно.
>>52371

>более одного анона, которые считают, что ты


Тащемта только ты и ещё один ворвавшийся в тред поборник категорий по Арнольду, который, видимо, скоро расскажет нам, что такое группоид.
Впрочем

>когнитивный диссонанс


>гонять нубов


>свои родные разделы, жж, лурк


Ты жертва Пикабу что ли? Так бы и сказал. В принципе это интересно, конечно, больше ебанатов с разных ресурсов, живее раздел, вот уже обсуждения какие-то. Правда довольно однообразные в твоём случае.
194 52383
>>52380

>скоро расскажет нам, что такое группоид



Под «нами» ты имеешь в виду «я и моя прелесть»? Ты же здесь один такой юродивый

подвести группоид под какие-нибудь преобразования едва ли должно быть хоть чуть-чуть трудно, если уж захочется; просто преобразования будут локального характера, и все
195 52384
>>52380
Про Арнольда ничего не знаю и в этом трэде отписывался давно по совершенно другой теме, но упомянутое выше определение группы действительно идейно близко к категорному мышлению. Это очевидно, думаю, всем, кто доказывал коммутативность более одной диаграммы в своей жизни. Со стороны именно ты выглядишь нелепо, честно говоря, игнорируя аргументы других и используя fallacies. Но мне так вы все тут долбоёбы.
196 52385
>>52383

>же здесь один такой юродивый


В том смысле, что опустился до разговора с тобой, что ли? И не с такими дебилами сталкивался.

>подвести группоид под какие-нибудь преобразования едва ли должно быть


Угу, можешь подавать заявку на грант уже, напишешь учебник по алгебре с позиций Арнольда, упразднив аксиомы группы и кольца окончательно. Безусловно, твой высер разделит судьбу двухтомника Ван дер Вардена и со временем я стану горд тем что имел честь общаться с тобой, тупым клоуном, на анонимной имиджборде. Ещё есть что сказать, или это всё?
197 52390
>>52384

>Но мне так вы все тут долбоёбы.


ты чо пёс ты чо?!!1
198 52401
>>52385
Наверно ты не заметил (да и вряд ли заметишь, хотя я тебе сейчас прямо на это укажу) у тебя самого давно не находится ничего сказать, кроме разнообразных оскорблений собеседников. Тем самым у меня есть сказать как минимум не меньше, чем есть у тебя. Поскольку, очевидно, ты сам останавливаться не будешь, не откажу себе и далее вставлять какие-нибудь реплики, если захочется
199 52485

>Я понимаю, что многие сейчас сначала узнают английские термины, типа "abelianization", а потом механически переносят их в русский язык. Давайте всё-таки безжалостно выкидывать таких монстров, как "абелианизация", и ставить нормальные давно существующие русские слова. В данном случае - абелизация.

200 52492
>>52485
Согласованность с английским важнее.
Да и чем abelization более русское, чем abelianization?
201 52494
>>52492
Так я-то согласен. Ну абелианизация, ну и что. Я, по крайней мере, только такое произношение слышал вживую. Меня это меньше коробит, чем "специальная" теория относительности или "линейная оболочка, натянутая на вектора".
Цитата, если что, из обсуждения на педивикии.
202 52554
>>52494
А что не так со специальной теорией относительности?
203 52557
>>52554
У него видимо частная теория относительности, из серии "категории частных и теория гомотопий".
204 57632
Читаю Акслера, SVD для оператора.

Прошу помощи с сингулярными значениями.

Каков смысл сингулярных значений? Собственные значения и вектора, например, определяют одномерные подпространства, инвариантные относительно оператора. То есть дают возможность разложить оператор. А сингулярные?

Как от SVD оператора переходим к SVD произвольного линейного отображения?
205 57639
>>57632

>А сингулярные?


Тоже самое, но теперь не только для эндоморфизмов.
206 57725
>>57632
Я всегда понимал что линейное отображение = поворот масштабирование другой поворот. Сингулярные значения - то самое масштабирование.

В догонку вопрос алгебраистам - интерпретацию svd в geometric algebra кто-нибудь ковырял. Есть где почитать чего?
208 57844
>>20 (OP)
Двач, мне нужен сайтец или архивы кровавой гэбни со всякими там примерами, логарифмами, уравнениями за все классы и сё такое.
Не хочется искать в поисковиках прост.
209 57845
>>57844
Скачай учебники за все классы
210 57847
>>57844
Купи Киселёва арифметику и алгебру. В трёх небольших книгах есть все необходимое
211 58119
>>49254
>>49255
Шафаревич
212 58124
>>52291
А где в комбинаторной геометрии топология или матлоге там?
213 58137
>>58124

>матлоге


В матлоге теорема о компактности, например.
214 60648
>>4726
Модульная арифметика, это такой пиздец, но я поясню вам:
Компьютеры не очень хорошо справляются с произвольно большими числами. Эту проблему можно решить, если выбрать максимальное значение и иметь дело только с числами, которые меньше максимума. Работает это как в часах с циферблатом и стрелками. Как перевести их, например, на 37 часов? Очевидно, разделить 37 на максимум — то есть 12 — и докрутить остаток. Так и здесь: любые вычисления, дающие результат больше максимума, мы «докручиваем» до числа в допустимом диапазоне.
215 60671
>>58119
а что он написал общую алгебру?
216 60677
>>60648
Ну не совсем с произвольными, а просто с рациональными. То есть любое деление они представляют в виде большой десятичной дроби. И поэтому сравнивают результаты деления взаимно простых чисел не с помощью знака равно а с помощью больше-меньше погрешности. Ты об этом?
Из за этого включается так называемая Floating-point arithmetics, которая даже не может правильно сравнить результат сложения двух десятичных дробей
0.1 + 0.2 == 0.3 // false
217 60679
>>60677
Приветствую, я паста. Вот мы и поговорили. https://xakep.ru/2019/08/27/elliptic-curve-cryptography/
218 60680
>>60677

>console.log(0.1 + 0.2 == 0.3); //false


Бле, внатуре. Чё за нах? Опять новый браузер ставить что-ли?
219 60681
>>60679
Аа я понял к чему ты клонишь. Могу сказать про это то что идея шифрования рса тоже имеет интересную историю, заключающуюся в том, что сначала был придуман принцип, по которому банк поставляет кейс, ключ от которого есть у банка, в который клиент кладёт свои данные и запирает, и много позже был придуман математический принцип с множителями, который смог воплотить его в жизнь.
220 60682
>>60681
Раз уж ты про RSA, держи это, моешь слить в zip: https://username1565.github.io/pgp/
После распаковки, работает локально, client-side на скриптах.

А вообще, эллиптическая криптография гораздо интереснее. Там ключи поменьше, а взломать так хуй.
Однако ECC ещё не завезли, потому что кодировать-декодировать сообщения в точки на эллиптической кривой - не понятно как.
Но у меня уже, по мере изучения, чё-то уже вырисовывается в мозгах, правда сформулировать толком не могу ещё.
221 60684
>>60680
Дело не в браузерах, просто конструктивная математика каличная.
222 60686
>>60684

>конструктивная математика каличная


В том смысле, что нет алгоритма, который бы мог два числа произвольных сравнить, поэтому если дробь не степень двойки, как 0.5, 0.25, 0.125 и тд, то там будет период, ну и на каком-то знаке после запятой округление(например 0.2 + 0.1 в двоичной это 0.0(0110) + 0.0(0011) и по идее должно получаться 0.0(1001) то есть 0.3, но где-то происходит округление и 0.2+0.1 = 0.30000000000000004 != 0.3 получается)
223 60717
>>60686

>конструктивная


>произвольных


/0
224 60734
>>60717
Произвольных вычислимых.
image.png136 Кб, 1059x125
225 61203
Ребят, вот вы в алгебре уже ебать прошаренные, а я только на пути возмужания. То, что сверху, как делать? Туплю, кажись уже передознулся этой алгеброй вашей. Понимаю, что f(a) в Fq имеет единственный ноль на a и что на остальных многочленах этой степени и ниже с коэффами из Fp на альфа не ноль, но что дальше делать -- не представляю.
Понимаю, что a порождает базис в факторкольце многочленов над Fp, в принципе если постараюсь, докажу, что элементы из Fp порождаются (хотя хз), но как линейные комбинации получить -- не представляю.
image.png4 Мб, 2048x1536
226 62643
как решить?
227 62673
>>62643
Ну ты хоть номер-то назови, ёпта, прояви элементарное уважение к сосачерам.
Если №10, то очевидная индукция + разложение по одной строке.
228 62674
229 62682
спасибо, очень остроумное решение из вики, надо потом будет через многочлены ещё порешать, ведь определитель то суть многочлен, следовательно можно равенство частей слева и справа проверить
230 62684
>>62682

>надо потом будет через многочлены ещё порешать


>можно равенство частей слева и справа проверить


Заебёшься. Типа ничего дельного из этого из этого не выйдет.
А если хочешь посмотрел доказательство, где в определителе используется то свойство, что он многочлен - смотри
https://ru.wikipedia.org/wiki/Определитель_Вандермонда
231 62698
>>62643
Хз. Это вообще решается??
232 62823
Сап,двач.Помогите с задачей,хуй знает что с ней вообще делать:найти число классов эквивалентности над С и над R квадратичных форм от n переменных.
233 62824
>>62823

> Сап,двач.Помогите с задачей,хуй знает что с ней вообще делать:найти число классов эквивалентности над С и над R квадратичных форм от n переменных.


Число классов эквивалентности - число видов нормированных диагональных матриц, соответствующих этим формам. Почему и сколько их сам догадаешься.
234 62825
>>62824
>>62823
Разве задача корректная, если не сказано, что за отношение эквивалентности?
235 62826
>>62825
Конечно, некорректная, но остальные инварианты вряд ли доступны в курсе алгебре мухосранскгу.
236 63026
>>20 (OP)
А которая из теорем де Рама неизвестна нормальным аналитикам?
237 63470
Двач помоги плз. Готовлюсь к сессии и читаю у себя в лекциях следующую хуйню:

Теорема. Для любой таблицы (1) интерполяционный многочлен существует в единственном виде. На (1) таблица инт. многочлена в общем виде
Доказательство:
Действительно, легко видеть что мн-н заданный ф-лой
(Тут формула форма Лагранжа)
это инт. мн-н в форме Лагранжа.

... (дальше единственность доказываем)

Почему так нахуй? Я не понимаю вообще откуда эта форма взялась. Подозреваю что препод разрешит это ей не рассказывать, но мне просто интересно. Как эту формулу нашли?
238 63475
>>63470
Я не совсем понял вопрос и за историю этой формулы не шарю, но, может быть, ты что-то осознаешь, если подставишь в эту формулу какой-нибудь из узлов xi и поймёшь, почему этот многочлен является интерполяционным.

Все слагаемые формулы, кроме i-го, обнулятся, потому что в числителе окажется множитель xi-xi. А в i-м слагаемом этого множителя не будет, зато все xi-xj в числителях и знаменателях сократятся, и в итоге i-е слагаемое окажется просто равным f(xi).
239 64392
Посоветуйте задачник к Винбергу,желательно с решениями.
240 64403
>>64392
Сборник задач по алгебре, Кострикин
241 64671
Анончик, хз куда написать.
Короче, в программе есть участок, где к разложенной по Холецкому матрице (LLT разложение) нужно прибавить другую, не разложенную матрицу R. Причем ответ тоже должен быть разложенный.
Есть вроде какое-то решение, и предполагает qr разложение R, но я не знаю нихуя, сложно пиздец. Спасибо.
242 64736
>>64671
Сверни разложенную матрицу, сложи и разложи результат снова.
243 64842
>>64392
Листки Независимого за любой год. Впечатление от Независимого как от места для матбогов неверное, не переживай. Проблема просто сдавать листки ПО ВСЕМ предметам, особенно если в своём вузе не проходишь эти предметы. Листки по алгебре адекватные.
244 66419
>>20 (OP)
Почему схему (понятие из алгебраической геометрии) можно назвать системой уравнений? Что такое схема?
245 66420
>>64842
А где-нибудь есть решения для этих листков?
246 66463
>>66419

>можно назвать системой уравнений?


Только локально, схема - локально окольцованное пространство, то есть, только ограничения пучка на открытые подмножества аффинны (те изоморфны спектру какого-то кольца).
15449613515740.jpg152 Кб, 1280x960
247 67141
>>64842
Листки нужно обсуждать, а если ты битард, то с кем?
248 67143
>>67141

>а если ты битард, то с кем?


Да давно бы уже запилили тут листкотред.
249 67163
>>60682

>кодировать-декодировать сообщения в точки на эллиптической кривой - не понятно как


Сообщение разбивается на блоки и каждый блок преобразуется в длинное число с системе считсления с основанием 256
250 67202
>>67143
Запили сам.
251 67257
>>67141
Пишешь любому НМУшному семинаристу по алгебре в соцсеточках/на мыло и просишь попринимать листки. Кто-нибудь да откликнется, особенно сейчас, когда все дома хуи пинают.
252 67297
>>67163
Это то же самое, что декодировать сообщение в байты.
Каждый байт - это цифра в 256-ричной системе исчисления. А дальше-то что делать с байтами?
В общем, ты некропостер ещё тот... Лол. Хорошо что я заметил твой ответ.
Я уже реализовал некое подобие эллиптической криптографии - тут:
https://github.com/username1565/mini_ecdsa/blob/master/ECC.py
253 67299
>>67297

>А дальше-то что делать с байтами?


Ну вот у тебя есть массив zhopa = { 0x01, 0x02, 0xFF, 0xDD } это твое сообщение. А дальше кручу-верчу наебать хочу, перегоняешь все это в длинное число примерно так, в зависимости от порядка байт (endianess): 01h x 256 + 02h x 256^2 + FFh * 256^3 + DDh x 256^4 = 123456789101112 (мне лень считать). И уже этот результат передаешь на вход криптосистемы, например, в виде точки (123456789101112, 1). Я уже забыл лекции по криптографии
254 67300
>>67299
Поправка: отсчет начинается с нуля, поэтому вместо 01h * 256 будет 256 в степени 0, то есть 1, получаем просто 01h
255 67319
>>67299
>>67300
То есть, ты, предлагаешь просто засунуть инфу в виде длинного числа - в x-координату, а y-координату точки - вычислить?
Прикол в том, что как видно отсюда:
>>67297

>https://github.com/username1565/mini_ecdsa/blob/master/ECC.py


на маленькой эллиптической кривой (y^2) mod 211 = (x^3 + 7) mod 211
с генераторной точкой (150, 22)
лежит всего 199 точек (198 + O), при этом 198 / 2 = 99,
и 99 точек имеют x-координаты одинаковые, а y-координаты - разные.
При этом, значения x-координат лежат в диапазоне [0, 211),
То есть не все числа от [0,211) являются x-координатами точек на кривой в конечном поле,
а лишь 99 чисел из этого диапазона.
То же самое, и для больших кривых, и для очень больших.
256 67603
Небожители, подскажите, из любого ли множества можно сделать группу при желании?
257 67641
>>67603
вот так:
https://ru.wikipedia.org/wiki/Свободная_группа

Стоит заметить ещё, указанная операция есть функтор, сопряжённый забывающему Группы -> Множества (см. Алуффи)
258 67828
>>67603
На любом непустом множестве можно ввести структуру группы, да. Достаточно ввести её на кардиналах. На конечном n можно рассмотреть Z\nZ. На бесконечном k - прямую сумму k экземпляров Z\2Z. Это множество последовательностей длины k из нулей и единиц, в которых лишь конечное количество элементов отлично от 0, с покомпонентным сложением по правилам 0+1=1+0=1, 0+0=1+1=0.
259 67984
>>67828

>на любом непустом множестве


>Достаточно ввести её на кардиналах


А у нас уже кардиналы множество образовывать стали?
260 68257
>>20 (OP)
Ты берешь 1000$ с собой к букмекеру, ставишь 1$ выигрываешь - ставишь ещё, проигрываешь - удваиваешь, до тех пор, пока не выиграешь

Какова вероятность того что я стану миллионером и того что проебу всю 1k$?
261 68260
>>68257
У тебя около 10 попыток, чтобы выиграть доллар.
262 68264
>>68260
1000 же, я забыл уточнить, вероятность 1\2
263 68265
>>68260
а ты про.. ну да
264 68273
>>67984
Чтобы ввести структуру на каждом из кардиналов, совсем не нужно объединять все кардиналы в какое-то единое множество.
265 68274
>>68273
Я почему-то подумал, что у тебя кардиналы будут элементами группы.
266 68366
>>68274
В принципе, подобные большие образования не запрещены. Например, класс всех множеств V является моноидом (очень большим) относительно операции объединения. Просто в ZFC такое сложновыразимо.
iKptTR429i.png36 Кб, 795x135
267 70622
Сап, двач
помоги решить задачу по гладким многообразиям.
Я пока рассуждал так. Допустим, L!=[L,L]. Тогда существует базисный вектор e такой, что он не равен коммутатору. Тогда можно получить, что его дуальная форма w имеет нулевой дифференциал, т.е. dw = 0. Препод задал встречный вопрос, является ли форма точной, т.е. существует ли функция f: w = df
268 70625
>>70622
Разве это не следует тривиально из того, что скобка есть дифференцирование (которое можно отождествить с каким-то гладким векторным полем)?
269 70631
>>70625
Может и следует, но мне это не очевидно... (
270 70644
>>70625
Или ты имеешь ввиду, что коммутатор векторных полей снова векторное поле?
Коммутатор есть производная Ли, но есть ли в этом толк?
271 70924
>>70622
Из односвязности следует, что первые когомологии де Рама нулевые. То есть, любая замкнутая 1-форма точна. В твоём случае это как раз и означает, что для w, где dw=0, найдётся f такое, что w=df.
image.png21 Кб, 849x281
272 75630
Что делаю не так? Корни же 1 и -6, тогда (1 + 6)^2 = 1 + 12 + 36 = 49
273 75648
>>75630
Корни 1/2 и -3.
274 75651
>>75630
Там же дискриминант просят найти лул
275 75652
>>75651
Блин, точно, что-то я напутал с корнями, 1/2 и -3, правильно. Бтв почему дискриминант? Разве разность корней - это дискриминант?
276 75653
>>75652
Квадрат разности корней это дискриминант, по определению.
277 75654
>>75653
ну, ещё умноженная на $a_n^{2n-2}$
278 77769
Помогите кто с алгеброй множеств
(A\B)\C=(A\C)\(B\C)
нужно доказать тождество используя алгебру множеств
image.png44 Кб, 1020x446
279 77781
ф.png78 Кб, 556x364
280 78684
Анон, я совсем отчаялся. Прошу, помоги разобраться с местом, подчеркнутым красным. Почему эти 2 утверждения равносильны?
281 78685
>>78684
Там нужно учитывать, помимо прочего, что (a,b)=1, a>0, b>0.
282 78690
>>78684
тоже не догоняю, хуйня какая-то, факт из ТЕОРЕМЫ(пиздец) проще доказать либо через табличку m на n, а лучше - через изоморфизм из КТО(ну типа надо показать что образ группы обратимых по умножению в Z/mnZ - это в точности группа обратимых по умножению в Z/mZ x Z/nZ, а порядок последней легко посчитать)

А блядь понял короче там опечатка: "взаимно прост с b" конечно. Ну на картинке какое-то нагроможденное доказательство через табличку, но без таблички, найди норм в инете.
283 78703
>>78690
БЛЯТЬ, Анон, СПАСИБО огромное, как я сам не заметил опечатку.
Можно даже подобрать числа, удов. условиям, например a=12, b=5, r=2, q=2 - получается, что 0<=r<b, 0<=q<a, (a,b)=1, (b,r)=1, значит, по этому долбаебскому утверждению, на попытку доказать которое у меня вчера ушло больше 2х часов драгоценного времени, получается, что (12, 12)=1, пиздец. Но это
я только теперь заметил. Спасибо еще раз.

Доказательство с табличкой видел, но скипнул, т.к. времени нет разбираться с новым доказательством. В итоге потратил еще больше с этой опечаткой, МДА.
284 79011
He увeрeн, что вопроc cтоит адрecовать этому трeду, но вcё-таки: имeют ли группы Ли приминeниe в физикe и ecли да, то в каких раздeлах?
285 79013
>>9328
cпаcибо оп
286 79020
>>79011
Конечно имеют, они фундаментальны в современной теорфизике
В частности, представления группы Лоренца (и группы Пуанкаре) очень тесно связаны с понятием элементарной частицы
Генераторы групп Ли это важнейшие операторы/наблюдаемые в квантовой механике
Операторы Казимира играют важную роль
Теорема Нётер конечно же
Вобщем, если одним словом описать развитие теорфизики в ХХ веке, то это будет 'симметрии', и группы Ли описывают непрерывные симметрии
287 80152
>>20 (OP)

>Если математика - царица наук, то алгебра - венец самой математики. Этот тред посвящён ей! ссым в нём на тех, кто занимается анализом и не знает теорему де Рама


А обязательно ли ссать на тех, кто занимается анализом?
288 80153
>>80152
если они занимаются анализом, но не знают теорему де рама, то наверно не помешало бы
289 80155
>>80153
А если знают?
290 80157
>>80155
за атью-зингера спросить тогда надобно
291 80158
>>79020

> Генераторы групп Ли это важнейшие операторы/наблюдаемые в квантовой механике


Расскажи пожалуста подробнее об этом
Серьёзно, неиронично
292 80160
>>80157
Это уже серьёзно. Это пятый курс по Мишиной программе?
293 80173
>>80160
Про индексы нужно любому чистому математику знать, это фундаментальная вещь
И тогда мы упираемся в пререквизиты и хуевые университетские курсы
Поэтому мы приходим к тому, что есть и в других областях - каждый разбирается только в своей ультра-узкой области и отрабатывает гранты высирая никому не нужные статьи про (H¨⇐N)-exact natural transformations arising from fibrant arrows over the co-hypercover of reflective bi-categories
294 80174
>>80173

>Поэтому мы приходим к тому, что есть и в других областях - каждый разбирается только в своей ультра-узкой области и отрабатывает гранты высирая никому не нужные статьи про (H¨⇐N)-exact natural transformations arising from fibrant arrows over the co-hypercover of reflective bi-categories


Арнольд кстати говорил, что разговоры про то, что сегодня уже ни один мощный математик не способен хотя бы общим пониманием охватить всю математику - это херня.
Типа проблема просто в том, что высираются тонны статей типа "об одном частном решении одного дифференциального уравнения" (примерно такая цитата была), которые одно по одному обсасывают.
Миша примерно так же считает, вроде. По крайней мере, насчёт ненужных статей.
295 80175
>>80174

>"об одном частном решении одного дифференциального уравнения"


Или статей об
"об одном топологическом пространстве и его гомологиях"
296 80176
>>80175
Да.
297 80178
>>80175

>"об одном топологическом пространстве и его гомологиях"


"об одном старом топологическом пространстве и одной его гомологии, посчитанной еще одним новым способом" - вот так надо!
298 80183
>>20 (OP)
Можно здесь задавать вопросы по алгебре? по моему моя проблема алебраического характера, в плане там простой матанализ
299 80184
Здравствуйте анончики.Я задавал вопрос в треде для новичков но там не ответили, поэтому задам здесь, если все же не ответят, пойду в mathoverflow

Проблема такова:
Вот есть функция f(u,v) она пока что абстрактная
Самое важное требование к этой функции - ассоциативность, то есть

f( f(u,v), w ) = f( u, f(v, w) )

Есть также дополнительные требования к ней во первых монотонное возрастание по u и по v

или иными словами df/du > 0 и df/dv>0

во вторых оно должно быть continuous and twice differentiable, не совсем понятно второе выражение но я полагаю возможность дифференциировать по первой переменной а потом по второй

Понятное дело, что даже с такими ограничениями подходящих функций бесконечное множество. Сам автор перед началом долгого поиска общей функции заранее говорит что если у вас есть любая invertible и монотонная функция G(u) то общим решением является
f(u,v) = G^-1( G(u)G(y) )

Я хотел убедиться в этом и решил взять arctan(u) как G(u). Она invertible и монотонная.Теперь имеем функцию f(u,v) = tan(arctan(u)arctan(v)), она ассоциативна
просто из за ее общей формы G^-1( G(u)G(v) ). Проверяю теперь ее монтонное возрастание по двум переменным

f(u,v)/du = tan'(arctan(u)arctan(v)) = 1/cos^2(arctan(u)arctan(v) ) ( arctan'(u)arctan(v) )= 1/cos^2(arctan(u)arctan(v) ) constant(v)
1/1+u^2

вроде бы при любых u производная от u будет строго больше нуля

и тоже самое будет со вторым

Теперь вопрос: почему когда рисую график этой функции tan(arctan(u)*arctan(v)) получается лютый кошмар, он вовсе не монтонен он скачет верх и вниз как бешенная. Почему так? Где мой прокол?
299 80184
Здравствуйте анончики.Я задавал вопрос в треде для новичков но там не ответили, поэтому задам здесь, если все же не ответят, пойду в mathoverflow

Проблема такова:
Вот есть функция f(u,v) она пока что абстрактная
Самое важное требование к этой функции - ассоциативность, то есть

f( f(u,v), w ) = f( u, f(v, w) )

Есть также дополнительные требования к ней во первых монотонное возрастание по u и по v

или иными словами df/du > 0 и df/dv>0

во вторых оно должно быть continuous and twice differentiable, не совсем понятно второе выражение но я полагаю возможность дифференциировать по первой переменной а потом по второй

Понятное дело, что даже с такими ограничениями подходящих функций бесконечное множество. Сам автор перед началом долгого поиска общей функции заранее говорит что если у вас есть любая invertible и монотонная функция G(u) то общим решением является
f(u,v) = G^-1( G(u)G(y) )

Я хотел убедиться в этом и решил взять arctan(u) как G(u). Она invertible и монотонная.Теперь имеем функцию f(u,v) = tan(arctan(u)arctan(v)), она ассоциативна
просто из за ее общей формы G^-1( G(u)G(v) ). Проверяю теперь ее монтонное возрастание по двум переменным

f(u,v)/du = tan'(arctan(u)arctan(v)) = 1/cos^2(arctan(u)arctan(v) ) ( arctan'(u)arctan(v) )= 1/cos^2(arctan(u)arctan(v) ) constant(v)
1/1+u^2

вроде бы при любых u производная от u будет строго больше нуля

и тоже самое будет со вторым

Теперь вопрос: почему когда рисую график этой функции tan(arctan(u)*arctan(v)) получается лютый кошмар, он вовсе не монтонен он скачет верх и вниз как бешенная. Почему так? Где мой прокол?
300 80191
>>80184

>twice differentiable, не совсем понятно второе выражение но я полагаю возможность дифференциировать по первой переменной а потом по второй


Неправильно полагаешь. Имеется в виду, что данная функция из класса $C^{2}$ то есть имеет все частные производные до второго порядка включительно.
301 80193
>>80191
понял тебя кстати я там производные посчитал не правильно, у функции tan(arctan(u)*arctan(v)) производные от обоих переменных не строго больше нуля

Но еще есть такой вопрос, вообще есть ли функция f(u,v) которая обладает строго положительными производными и также обладает ассоциативностью f( f(u,v), w ) = f( u, f(v, w) )?

На уме только f(u,v) = u + v
302 80196
>>80178
Ахаахахаа
303 80200
>>80193
арктангенс до pi/2
пронормируй, чтобы был до 1
и будет тебе щасье
304 80244
>>80200
анон я туплю страшно
мне надо поделить arctan(u) на pi/2?
тогда будет функция tan((arctan(u))/(pi/2)*(arctan(v))/(pi/2))

как это сделает производные строго позитивными? я тупой
305 80246
>>80244
tan((pi/2)(arctan(u))/(pi/2)(arctan(v))/(pi/2))
там чуть сокращается, конечно

у тебя уже всё есть
производные будут какие надо при аргументе до pi/2

ты сам написал hint к решению, но не до конца понял.
его идея в том, что можно "деформировать" множество, на котором задана операция, при помощи любой обратимой функции (сохраняющей то, что нам надо сохранить по условию - порядок, непрерывность, гладкость)
306 80265
>>80246
но анон

смотри, производная функции tan((pi/2)(arctan(u))/(pi/2)(arctan(v))/(pi/2)) от u

f(u,v) = 1/cos^2( arctan(u) arctan (v)/(pi/2) ) arctan(v)/(pi/2) * 1/(1+u^2)

тут u не может сделать значение функции негативным, но все портит константа arctan(v)/(pi/2), если она негативна а она, негативна если v от минус бесконености до нуля, то производная вся тоже больше нуля

вопрос: причем тут нормализация?
307 80268
>>80265
меньше нуля
308 80269
>>80265
тут u не может сделать значение функции негативным, но все портит константа arctan(v)/(pi/2), если она негативна а она, негативна если v от минус бесконености до нуля, то производная вся тоже меньше нуля*
309 80276
>>80269
даже когда просто x*y то, внезапно, одна из производных тоже может быть меньше нуля, когда x<0 или y<0.
что там с условием задачи?
310 80304
Подскажите, какой нейтральный элемент для операции "тетрация"? И как правильно сказать, например, 3 в степени 4, но для этой операции, а не для возведения в степень?
311 80316
>>20 (OP)

>Вектор - направленый отрезок с точностью до параллельного переноса


>с точностью до параллельного переноса


Пожалуйста объясните последнюю фразу.
312 80317
>>80316
это означает, что все направленые отрезки, которые можно получить друг из с друга с помощью до параллельного переноса, считаются одним и тем же объектом

т.е. операция параллельного переноса не изменяет твой объект
313 80321
>>80318
Два параллельных одинаково направленных отрезка одинаковой длины представляют один и тот же вектор
314 80338
>>80321
Понял.
Анон, а можно уточню про базис: правильно ли я понимаю, что любые 2 непропорциональные векторы будут базисом R2?
Работает ли такое же для любого пространства (то есть любые N непропорциональные векторы базис N-мерного пространств)?
315 80340
>>80338

>любые 2 непропорциональные векторы будут базисом R2?


правильно

>Работает ли такое же для любого пространства (то есть любые N непропорциональные векторы базис N-мерного пространств)?


представь себе R^3, горизонтальную плоскость и любые три вектора на ней. а теперь нарисуй вектор, торчащий из этой плоскости вертикально вверх, можно его через них выразить?
316 80341
>>80340
Благодарю.
317 80400
Что значит вертикально? Может случится так что привычного угла в 90 градусов эти векторы не дают, и в этом базисе вертикального вообще ничего нету
318 80403
>>80400
угол между векторами - это дополнительная характеристика, которая определена только в пространствах со скалярным произведением. про базисы можно говорить, не упоминая углы вообще (работая в линейных пространствах, в которых скалярное произведение не определено)
image.png14 Кб, 650x156
319 81803
Не понимаю, как нужно рассуждать, чтобы прийти к двум случаям. Как бы если проверить, то да, все правильно, но когда ты встречаешь такое уравнение, то не понятно куда дальше крутить
320 81804
>>81803
ну как варианты рассуждений:
1. или общий множитель равен нулю, или на него можно поделить
2. просто перенести все налево и разложить на множители
321 81805
>>81803
P.S. не знаю, что вопрос про тригонометрические уравнения делает в треде алгебра, но это, вероятно, из-за того, что школьный предмет, который по сути на 90% начала анализа (непрерывные аналитические функции, действительные числа и операции с ними, ...) называют почему-то алгеброй
322 81809
>>81805
Какой должна быть правильная школьная алгебра?
323 81810
>>81809
главное - тригонометрия должна быть исключена полностью

это какой-то дикий атавизм, неприемлемый вообще
324 81811
>>81803
Домножь на sin6x
(cos(6x)cos(2x))sin6x=cos6x
Если cos(6x)!=0, то на него можно поделить, и получим cos(2x)sin(6x)=1. Если cos(6x)=0, то равенство так же выполняется.
>>81805
Для решения школьных уравнений не нужно ничего из анализа. Это та же школьная алгебра, манипулирование значками, просто вместо qx пишут cos(qx), вместо формул сокращенного умножения формулы сумм углов и тд.
325 81812
>>81810
ну полностью тригонометрию исключать нельзя - оставить ее с большим уклоном в геометрию и физику

я бы
1. вернул бы комплексные числа, может быть как раз через них и рассказал тригонометрию в курсе алгебры (в геометрии пусть будет как есть)

2. добавил бы введение в группы - ну это просто, интересно и красиво

3. можно добавить немного про многочлены типа локализации корней, раз уж говорим про комплексные числа

4. можно попробовать рассказать про другие алгебраические структуры - жаль, до Галуа вряд ли получится дотянуть

5. анализ рассказал бы описательно - выкидываем нафиг всю главу про пределы, производную вводим как скорость - через махание руками, так же и про интегралы. вметсо этого хоть как-нибудь постараться впихнуть что-то про ряды - опять же для физики и без доказательств

ну п. 5, конечно, не в тему треда

Короче, после того как детей научили считать, я бы превратил курс математики в описательный типа природоведения, без строгих доказательств. Вроде того как в школе про теорию относительности и квантовую физику рассказывают - чисто красивые факты, чтобы интерес развивался.
326 81813
>>81812

Мне нравится, как чаще всего вводят разные тригонометрические формулы, типа вот смотрите косинус-синус и вот такие правила выполняются, без доказательства, без нихера. Потом заставляют их формально применять. Какой в этом смысл? Да абсолютно никакого, через год это абсолютно начисто забывается.
327 81815
>>81812
не знаю, зачем школьникам ряды и группы
а вот про комплексные числа рассказать дело хорошее
имхо
92342e58a6ca9ddabd6bdf18dbd8024b.jpg72 Кб, 500x733
328 81823
Лично я против уравниловки. Не бывает сферического школьника в вакууме. Может быть так, что один человек с трудом осваивает понятие доказательства и не может складывать дроби, а другой в состоянии считать когомологии, причем и возраст этих людей одинаков, и место жительства совпадает. Зачем учить этих людей одному и тому же одинаковыми способами? Очевидно, что им нужно разное обучение. Универсального математического знания, нужного всем гражданам поголовно, просто не бывает. Вместо классов средней школы, напоминающих конвейер какой-то абсурдной фабрики по производству человеческой биомассы, нужно делать небольшие кружки по интересам с разными наборами изучаемых тем. И эти кружки пусть будут рассчитаны на разных людей.

Какие-то люди, само собой, выберут никуда не ходить и ничему не учиться. Ну и пусть, это их дело. Такие люди и в общеобразовательной школе ничему не учатся. Они там и сами страдают, и остальным мешают, сковывают чужие таланты.
329 81826
>>81823

>Зачем учить этих людей одному и тому же одинаковыми способами?


$$$
нанимать вдвое или втрое больше преподавателей/администрации/уборщиков, арендовать или выкупать новые помещения для школ, составлять разные программы, и т.д., это всё деньги
если с одних налогов это финансировать, то non-sustainable очевидно, отсюда и частные школы
ты должен был это проходить классе в 9-ом на уроке условного обществознания
если постарше, то можешь почитать серьёзные экономические статьи (как, кстати, хороший пример применения базового матанализа, гладкости, выпуклости, и проч.), где налогообложение обсуждается с точки зрения deadweight loss и прочих externalities
если рассуждать идеалистически, то конечно материально-одарённые государства (например, полезными ископаемыми, ну понятно кто имеется ввиду) должны бы были все эти лишние деньги вкладывать в пенсионное обеспечение, здравоохранение, и образование, а не в ебаные тaнки и прочую хуйню (например то же ОАЭ расписало свой мега-план траты нефтебаксов на образование, науку, и проч вплоть до колонизации марса в 2100 году)
но что есть то есть
330 81827
>>81812
В школьной математике есть пропасть между античной математикой и математикой 18 века.
Лучше программу забить средневековой математикой. Она и нетривиальная, не слишком уж сложная, и полезная. Логарифмы, задачи на максимум и минимум и т.п. Короче всем что позже убил, основанный на этих вещах, матанализ.
331 81834
>>81826

>проходить классе в 9-ом


У меня был экстернат. На всякую ерунду тратить время не пришлось.
332 81835
>>81823
Какая жуткая картина. Эмоции охуенно переданы. Дед будто с плаката слезет и орать начнёт. Есть ещё?
333 81839
>>81835
ты совсем из ебеней, что ли?
погуглил бы хоть, я не знаю

какие же зумеры дегенераты
334 81842
>>81835
Правда не знаешь? Это же пинк флойд.

https://youtu.be/YR5ApYxkU-U
335 81851
>>81839
>>81842
Почему старичье считает, что зумеры обязаны знать уважать то говно мамонта, которое было популярно десятки лет назад? У меня один знакомый дед называет темными людей, которые не угадывают, из какого совкового фильма он в очередной раз произнес цитату.
336 81853
>>81851

>один знакомый дед называет темными людей, которые не угадывают, из какого совкового фильма он в очередной раз произнес цитату.


это действительно так. Это называется культурный уровень. Нормальные математики такие как Вербицкий или Громов очень много знают помимо своей специальности. А узкие специалисты с кругозором зубочистки нужны лишь ушлым капиталистам, чтобы люди не бухтели и сидели в своей яме.
337 81854
>>81851
Потому что это так называемое говно мамонта, золотой век культуры.
338 81855
>>81851
а что они должны уважать? тем более в данном случае зумер даже поленился погуглить, хотя происхождение восхитившей его картинки прямо на ней написано

>один знакомый дед называет темными людей, которые не угадывают, из какого совкового фильма он в очередной раз произнес цитату.


правильно делают, тащемта.

>>81854
+
339 81856
>>81855
Считать чем-то равноценным советскую культуру и нормальную всё-таки не следует. Есть мнение, что во многих случаях там та же ситуация, что с The Winner Takes It All и Позвони мне, позвони.
340 81857
>>81856
стоит

>во многих случаях там та же ситуация, что с The Winner Takes It All и Позвони мне, позвони.


культура это палимпсест (c)
341 81858
>>81857
Именно поэтому в российской науке (и в алгебре в частности, раз уж мы в алгебра-треде) так распространен переводной плагиат. Ачотакова.
342 81859
>>81858

>так распространен переводной плагиат. Ачотакова.



в первый раз слышу
но если даже и так, то вообще ничего, ящитаю
343 81860
>>81854

>золотой век культуры


Ты сказал?
Музыка того же Варга, периода когда он сидел в тюрьме и имел лишь синтезатор, намного качественней, чем вся та дрисня из 60х. Исключений единицы, типа малинового короля.
344 81863
>>81860

>Музыка того же Варга <..> намного качественней, чем вся та дрисня из 60х



нет
"музыка варга" вообще довольно примитивная, во все периоды
346 81956
>>9299

> Ну и для того, чтобы понимать, что говорят алгебраисты, когда они выёбываются.


Для того, чтобы это понять, одного учебника (и даже двух) недостаточно

>Посоветуйте


Для начинающих - Hungerford, например, или Винберг

>Очень желательно с применением полученных знаний на практике


Хорошо, а причём здесь алгебраисты? Алгебраистам, как и всем математикам, на приложения похую.
Попробуй Gallian, по-моему там было что-то прикладное.
Или ещё Abstract Algebra with Applications

>о бросил потому что не понял, нахуя вообще нужно.


Странно, учитывая что эти вещи как раз-таки прикладываются и используются на ура, в отличие от каких-нибудь резольвент модулей
Группы и теория представлений - это ключевые компоненты теорфизики уже лет 100 как
Группы ещё используются в химии и программировании, например
347 81983
>>9299
Группы дохуя где применяются, приложения в какой области тебе интересны?

Так или иначе, сначала разберись с теорией. Читай Кострикина или уже предложенного Винберга.
348 83829
Здарова работяги, нужно за ночь выучить алгебраические структуры, рац.дроби и многочлены на уровне первого курса, погнали нахуй
349 83830
>>83829
гони.
мне вот ничего не надо, буду сидеть и резаться в видеоигры
350 83836
>>83829
Какие именно структуры?
Вон двумя постами выше рекомендации для самых маленьких.
351 84029
>>81956
Для начинающих David Lay Linear Algebra with applications, там даже школьники вывезут
352 84030
>>20 (OP)
Сап двач, если я вообще не вывожу анализ но кайфую от алгебры, это нормально? Или чтобы замастерить второе нужно обяз ахуенно шарить в первом
353 84040
>>84030
Мне кажется, абсолютно нормально
Вот не знать топологию и кайфовать от алгебры — это, по-моему, какой-то аутизм, а анализ не знать норм
354 84069
Почаны, пусть я доказал основную теорему алгебры. Тогда легко понять, что $(Z/pZ)^ \cong Z/(p-1)Z$.
Пусть стоит задача отыскать образ и ядро такого гомоморфизма(очевидно) $f \colon (Z/pZ)^
\to (Z/pZ)^*$, $f(n)=n^{10}$.
Возникает эвристика: а давайте смотреть не на данный гомоморфизм, а на такой: $\tilde f \colon Z/(p-1)Z \to Z/(p-1)Z$, $\tilde f(n)=10n$.
Для последнего легко найти образ и ядро, ну значит такие же образ и ядро у исходного гомоморфизма. Но как это формально доказать, т.е. что $Im \tilde f \cong Imf$ и $Ker \tilde f \cong Kerf$?
Че-то туплю, мб диаграмму какую-то нарисовать?
355 84071
Почаны, пусть я доказал основную теорему алгебры. Тогда легко понять, что $(Z/pZ)^ \cong Z/(p-1)Z$.
Пусть стоит задача отыскать образ и ядро такого гомоморфизма(очевидно) $f \colon (Z/pZ)^ \to (Z/pZ)^$, $f(n)=n^{10}$.
Возникает эвристика: давайте смотреть не на данный гомоморфизм, а на такой: $\tilde f \colon Z/(p-1)Z \to Z/(p-1)Z$, $\tilde f(n)=10n$.
Для последнего легко найти образ и ядро, ну значит такие же образ и ядро у исходного гомоморфизма. Но как это формально доказать, т.е. что $Im \tilde f \cong Imf$ и $Ker \tilde f \cong Kerf$?
356 84104
>>84040
Имеется в виду математический анализ? Я хочу в алгебру вкатиться, но в калькулусе я полный ноль.
357 84116
>>84104
можно вполне изучать алгебру, не зная калькулюс
358 84117
>>84116
Сначала матанализ, а алгебра потом.
359 84118
>>84117
Можешь аргументировать? Планирую пройти начала множеств Шеня и штурмовать алгебру, а тут ты говоришь, что надо сначала матан заботать.
360 84119
>>84118
в алгебре не используется мат. анализ
анон>>84117 может аргументировать как угодно, но это будет отсебятина
361 84121
>>84118
Алгебра нужна только для расширения матанализа на размерности выше трёх.
362 84122
>>84121
Так я хочу чисто алгеброй заниматься, а не матанализом.
363 84132
>>84121
Скажи еще что алгебра нужна чтобы генерализировать решение линейных уравнении для R^n
364 84133
>>84132
еще приведение квадратичных форм к каноническому виду, второе высшее достижение алгебры
365 84143
>>84133
при этом математический смысл сего действа ты вряд ли расскажешь
366 84185
>>84143

>при этом математический смысл сего действа ты вряд ли расскажешь


Не он, но я даже пытаться не буду, потому что на 99% будет подъёб про какую-нибудь "жутко полезную и фундаметнальную" хуйню вроде следа в расширении поля, когомологий Понтрягина, или скобки Уайтхэда, в которых "на самом деле" проясняется смысл приведения формы к каноническому виду

Не удивлюсь, если на шиз-нкатлабе есть 10тистраничный опус о связи канонического вида с (∞,1)-категориями
367 84192
>>84185

>потому что на 99% будет подъёб про какую-нибудь



ой, да нет же: просто замена базиса на базис, составленный из собственных векторов. ничего сверхъестественного

тоже не>>84143
368 84196
>>84192
удобно потенциальную энергию представлять, вторую вариацию, формы колебаний, для нас прикладников это ценно
369 84291
>>84118
Зачем Шень, если ты хочешь только алгебру изучать? Шень будет в тему перед анализом или топологией, но не обязателен.
370 84312
>>84291
Разве для алгебры множества не нужны?
371 84319
>>84312
Не на таком уровне, как у Шеня.
372 84325
>>20 (OP)
Где достать части 2-6 курса Вавилова? Получается нагуглить только первую.
374 84372
Аноны, помогите. Половину лекций проебал и вообще не понимаю что происходит. Экзамен через 2 дня. Как к этому вообще подготовиться
375 84374
>>84372
Вот ещё
376 84375
>>84372

> Половину лекций проебал и вообще не понимаю что происходит.


> помогите


Каким образом можно вложить материала объемом в половину, хотя кого ты наебываешь малыш. Ты проебал вообще все лекции и не появлялся в вузике. До осени, до пересдачи, у тебя будет маса времени.
377 84381
>>84375

> Ты проебал вообще все лекции


Половину да, половину не понял наполовину

> не появлялся в вузике.


Меня бы и не пустили. Короновирус хуле
378 84384
>>84381

>Половину да, половину не понял наполовину


До осени думаю сможешь осилить.
379 84386
>>84384
Я просил помочь, а не нахуй послать. Мне хотя бы темы по номерам узнать чтоб найти аналогичные примеры
380 84389
>>84386

> Мне хотя бы темы по номерам узнать


У тебя совсем какие-то проблемы? На твоих пиках, 1 номер билета, в нем название тем.
381 84391
>>84386

Ну ты спросил типа, как мне нихуя не делать, дрочить-бухать и все сразу уметь и знать? Ответ "пошел нахуй" вполне уместный, бро. Никак.
382 84393
>>84372
>>84374
Не по теме треда

Ну если только притянуть вопрос "найти корни уравнения" к выбору кольца
383 84430
>>84375
Сдал на 4
384 84439
>>84430
ну списал или еще что, это полезный скилл, крутиться.
385 84440
>>84430
а теперь пошёл нахуй
386 84443
>>84439
На самом деле препод немного помогал
>>84440
Ок. Всем удачи.
387 84485
Увидел посты про алгебру Клиффорда и тоже решил почитать.
Если какая-то мотивация/интуиция у идеала, по которому мы берём фактор в определении алгебры? Вот например во внешней алгебре мы берём фактор по идеалу [math] x \otimes x [/math] и это легко мотивировать (если мы хотим построить что-то, что "измеряет" площади, то "коллинеарные" объекты будут схлопываться в нуль).

В определении алгебры Клиффорда (с билинейной формой B) можно рассмотреть два идеала (которые совпадают, конвенции знака бывают разные):
1) [math] x \otimes y + y \otimes x = 2B(x,y) [/math] - знаю, что похожая штука появилась у Дирака при попытке разложить оператор Клейна-Гордона. Но всё-таки алгебра Клиффорда видится более фундаментальной (да и появилась раньше), так что это так себе мотивация;

2) [math] x \otimes x = B(x, x) [/math] - уже получше и поестественней. Мы хотим, чтобы наше новое умножение схлопывалось в скалярное для y=x. Но.. почему?

Я уже почитал вперёд и увидел, что из одной только этой аксиомы (ну и ассоциативности/дистрибутивности) можно вывести всю алгебру, и как много чего (вроде C и H) красиво вкладывается. Я полезность под вопрос не ставлю. Из моего опыта, если мотивация "так будет удобно в дальнейшем", то я просто что-то не понимаю из других областей или не знаю исторического контекста.

У самого Клиффорда в статьях определяется по другому (через фактор определяли в 50х Бурбаки и Шевалле).

Так есть какая интуиция/интерпретация у идеала #2) вроде простой интерпретации идеала для внешней алгебры?
388 84486
>>84485
все определения хорошо задавать инвариантно, то есть через универсальные свойства и потом из этого выводить другие равносильные определения типа вот таких явных построений. Посмотри тут какое универсальное свойство алгебр клиффорда: https://en.wikipedia.org/wiki/Clifford_algebra#Universal_property_and_construction.
И в конце говорится вот это Namely, Cl can be considered as a functor from the category of vector spaces with quadratic forms (whose morphisms are linear maps preserving the quadratic form) to the category of associative algebras. The universal property guarantees that linear maps between vector spaces (preserving the quadratic form) extend uniquely to algebra homomorphisms between the associated Clifford algebras.
так что всё очень естественно
389 84506
>>84486
Это всё интересно (хоть и повторяет изложенное в моём теоркат-ориентированном учебнике), но не отвечает на мой, совершенно конкретный, вопрос совершенно. Впрочем, я уже нашёл ответ в другом учебнике для хлебушков.
390 84522
Товарищи хелп, реквестирую инструкцию по приведению квадратичной формы к нормальному жордановому виду(максимально для тупых), калькуляторы при проверке результатов выдают хуйню.
391 84526
>>84522
матрица квадратичной формы диагонализуется (приводится к диагональному виду)

для этого ищешь собственные вектора матрицы для твоей формы, затемх их ортоганализуешь
392 84531
>>84526
диагонолизовать и найти собственные вектора вообще без проблем, а ортогонализовать как?
393 84535
>>84531
Cобственные векторы, соответствующие разным собственным значениям ортогональны друг другу.
394 84541
>>84535
а если они отвечают одинаковым с.з., то необязательно
395 84548
Ребят, решил вкатиться в топологию. С горем пополам более- менее осиливаю. НО, когда приступил к гомологиям. Понял, что не понимаю изоморфизм, гомоморфизмы. Да и вообще усомнился в моём понимании фактормножеств. В связи с чем, прошу у уважаемых анонов каких - нибудь задачников с РЕШЕНИЯМИ по этим темам. Да и по гомологиям было бы неплохо, но это, веоятно за гранью фантастики.
396 84559
>>84535
а если собственное значение одно, у него 2 собственных вектора. Далее характеристический многочлен в ноль идет и у него базис ijk палучаеца. Допустим один вектор я беру из собственных, второй из ijk, а третий откуда брать, чтобы матрицу перехода получить?
397 84560
>>84559
я не очень понял, что ты хочешь сказать.
у диагонализуемой матрицы всегда столько линейно независимых собственных векторов, какова размерность пространства
на то она и диагонализуема
398 84669
Какая ирония, Куммер (идеальные числа которого легли в основу понятия идеала) не смог понять идей Грассманна (алгебра которого строится как фактор по идеалу), дал негативную рецензию на его статью, из-за этого Грассманна не взяли преподавать в университете и в конечном итоге Грассманн бросил математику и стал видным лингвистом в санскрите.
Клиффорд, который один из немногих догадался, как связать алгебру Грассманна и алгебру Гамильтона, не смог в сбалансированный распорядок дня и рано умер от туберкулёза, оставив нам полторы странички про свои идеи.
В итоге имеем монстра Гиббса-Хевисайда, которого до сих пор преподают аж в школе нахуй, вместе с неассоциативным выкидышем произведения Клиффорда/Грассманна.

Чем больше читаю про историю алгебры, тем больше понимаю, сколько я не понимаю.
399 84683
>>84669

>Куммер


Смешное имя
image.png303 Кб, 480x333
400 84717
Господа, поясните за этого джентльмена ньюфагу.

Пару раз смотрел его ролики, не впечатлило, ну нормально так обьясняет, многовато болтовни. Но потом...как я понял его странности, а то есть его непринятие действительных чисел, мне стало крайне странно смотреть его математику.

1) Кроме его странностей, насколько полезны его видео?
2) Как вы относитесь к его идеям, имеено к его отношению к дейст числам. ?
401 84722
>>84717

>1) Кроме его странностей, насколько полезны его видео?


Если тематика не сильно связана с его шизой - то видео как видео. Например, по вводному алгтопу или проективной геометрии.

>2) Как вы относитесь к его идеям, имеено к его отношению к дейст числам. ?


Так же, как и 99.999% остальных математиков. Основная проблема даже не в его позиции, а в её популяризации - он-то базовую математику знает сносно, а вот типичные слушатели - нет. Он просто деньги же делает.
Screenshot 2021-07-07 at 15.19.33.png24 Кб, 476x177
402 85319
>>84326
Уточню, я искал книги Вавилова по линейной алгебре
403 85325
>>85319
Вторая часть вроде издана в бумаге издательством ОЦЭиМ СПбГУ в 2006 году (232 страницы) и не существует в электронном виде, про остальные части вообще ничего не известно. Можешь попробовать написать самому Вавилову, других вариантов походу нет. А будь бы в России свой Спригер, то возможно такой хуйни с поиском книжки изданной хуй-знает-когда хуй-знает-кем не было.
404 85328
>>84325
Нигде. Они не написаны, существуют только в мыслях автора.
405 85626
>>85328
А есть ссылка на финальную, ну или хотя бы самую последнуюю версию pdf'ки Вавилова "не совсем наивная теория множеств"? То, что с ходу гуглится, какое-то совсем сырое и не законченное.
406 85627
>>85626
Хз, попробуй самому Вавилову письмо написать.
407 90473
Тред мёртвый, конечно, но, надеюсь, тут кто-то ещё сидит и готов помочь неразумному.

Возник один вопрос, который связан с разделом алгебры, в который я почти не заглядывал (пусть это и основы).

Проблему свою оформлю в несколько постов.

Первый пик — это вопрос, который возник у меня.
Второй - это ответ, который мне дали.
408 90474
Вот то (1-3 пики), на что в ответе ссылаются под here.
Последний пик — это изначальное утверждение, которое я увидел.
409 90475
>>90473
1) $\mathbb{Q}(X)(x_0)$ - это поле частных (дробей, составленных из двух полиномов с рациональными коэффициентами, где переменная принимает значение в $X\sqcup \{x_0\}$. Трансцендентный базис $\mathbb{R}$ над $\mathbb{Q}$ по определению это такое множество $A=X\sqcup \{x_0\}$, что $\mathbb{R}$ алгебраично над $\mathbb{Q}(X)(x_0)$.
2) Потом, кажется, так как в любой точке из $X\sqcup \{x_0\}$ $\mathbb{Q}(X)(x_0)$ принимает значение в $\mathbb{R}$, то $\mathbb{Q}(X)(x_0)\subseteq \mathbb{R}$, поэтому на $\mathbb{Q}(X)(x_0)$ можно задать дифференцирование, которое потом можно продлить на всё $\mathbb{R}$ единственным способом.
3) Нетривиальное дифференцирование в области целостности можно расширить на поле частных, поэтому дифференцирование можно рассмотреть просто на кольце многочленов с рациональными коэффициентами, где переменная принимает значение в $X\sqcup \{x_0\}$.

Если всё вышеперечисленное мною понято правильно, то мне непонятно, почему коэффициента там из $\mathbb{Q}(X)$, а не из $\mathbb{Q}$.
410 90587
>>90475
Потому, что в лемме "here" предполагается, что L алгебраично над k (если применять её в нужном тебе направлении (1)=>(3)). По этой причине взять k=\mathbb{Q} и L=\mathbb{R} не получится.
411 90595
>>90587
Но ведь $\mathbb{Q}(X)(x_0)$ — это же дроби, где наверху и внизу многочлены с коэффициентами из $\mathbb{Q}$, а значениями переменной в $X\sqcup \{x_0\}$, если я правильно понимаю. Казалось бы, что многочлены на последнем шаге мы тоже должны рассматривать с коэффициентами из $\mathbb{Q}$.
412 90603
>>90595
Понял тебя, кажется. Ты хочешь думать о \mathbb{Q}(X)(x_0) как о поле частных кольца \mathbb{Q}[X\sqcup \{x_0\}], а не кольца \mathbb{Q}(X)[x_0]. Тогда при построении ненулевого дифференцирования D нужно будет описывать как оно действует на многочлены с коэффицентами из \mathbb{Q}, как ты хочешь, но с переменными из X\sqcup \{x_0\}, вместо многочленов с коэффициентами из \mathbb{Q}(X), но с одной переменной x_0, как тебе написали. Видимо, можно и так тоже. По линейности и по правилу Лейбница достаточно определить D на каждом элементе из X\sqcup \{x_0\}. Например, D(x_0)=1 и D(x) = 0 для всех x\in X.

Кстати, я ранее неверно написал, что тебе нужно (1)=>(3). Нужно (1)=>(2).
413 90605
>>90603

>а не кольца \mathbb{Q}(X)[x_0]


Хм, я просто думал, что по определению трансцендентного базиса $A$ расширения $L/K$ у нас $L$ должно быть алгебраично именно над $K(A)$. А в данном случае у нас ведь $A=X\sqcup\{x_0\}$. Или тут как-то можно совершить переход от частных кольца $\mathbb{Q}[X\sqcup \{x_0\}]$ к частным кольца $\mathbb{Q}(X)[x_0]$? Я просто в этой области только несколько определений знаю, не более.

Хотя они, наверное, изоморфны ведь, как, например, $\mathbb{Q}[x]$ изоморфно $\mathbb{Q}[a]$ для любого выбранного $a$, поэтому так сделать можно.
414 90607
>>90605

>Хотя они, наверное, изоморфны


Да. Оба поля изоморфны $\mathbb{Q}(X\sqcup \{x_0\})$. Можно, например, воспользоваться универсальным свойством поля частных. Более наивно, отношение $\frac{p}{q}$, где $p, q\in \mathbb{Q}[X\sqcup \{x_0\}]$ можно, "собрав коэффициенты" перед степенями $x_0$ представить себе как отношение элементов из $\mathbb{Q}(X)[x_0]$, и аналогично в обратную сторону.

>например, Q[x] изоморфно Q[a] для любого выбранного a


Только если $a$ трансцендентно над $\mathbb{Q}$. Иначе, $Q[a]\simeq Q[x]/(p(x))$, где $p(x)$ - минимальный многочлен для $a$.
415 90609
>>90607
Спасибо, анон. Это как раз то, что мне нужно.

Я, вообще, приятно удивлён, насколько алгебраическая теория чисел красивая, хоть и заглянул туда совсем чуть-чуть. Раньше-то я думал, что это что-то наподобие рамануджанщины.
2021-12-2102-58-29.png96 Кб, 963x417
416 91207
ОТКУДА БЕРУТСЯ ЭТИ ШИЗОВЫЕ ФОРМУЛЫ В АНГЕМЕ?
Ну вот например пик(общий перпендикуляр к двум прямым), векторное произведение понятно - ищем перпендикуляр, почему в первой строке какая то хуйня? почему во второй строке направляющий вектор?
Есть чувство что эти формулы довольно просто выводить если понять как это делать, но этого я еще не понял. Что почитать чтобы понять как составлять такие системы для разных случаев, в зависимости от того что ищем?
417 91210
>>91207

>Есть чувство что эти формулы довольно просто выводить


все эти формулы выводятся по определению через разложения векторов по базису. что именно происходит в твоём случае, понять нельзя

общий перпендикуляр к двум прямым (если речь идёт о векторе) даётся векторным произведением направляющих векторов

выписываем эти вектора (их разложение по базису),
вычисляем векторное произведение (есть формула)
получаем ответ
418 91212
>>91210
на пике просто пример
мне интересно как в целом выводятся разные формулы в ангеме, что почитать чтобы понять как их правильно раскладывать, какие системки составлять?
419 91213
>>91212
ещё раз, эти формулы получаются напрямую из определений путём (несложных) вычислений; основной инструмент - представление участвующих в вычислении векторов через координаты; потом и результирующие формулы получаются в координатах
420 91241
>>91212

>как в целом выводятся разные формулы в ангеме


Ну как-то так https://youtu.be/zWMTTRJ0l4w
421 91243
>>91241
годно, грац
422 95524
Лучше Алуффи уже что-нибудь появилось за 13 лет? На чем сейчас илитно вкатываться? Чтоб с аджоинтами и модулями, но как для продвинутых первокуров.
Screenshot20220501-174029~2.png266 Кб, 1080x1002
423 95526
>>95524
Почему у Алуффи определитель даётся формулой?
424 95530
>>95526
так это определитель матрицы, какая там формула быть должна?

всякие функторы внешней степени и прочая ерунда - это всё относится к линейным операторам, а не к матрицам.
425 95564
>>95524

>Чтоб с аджоинтами и модулями, но как для продвинутых первокуров.


Нахуя, если потом все группы порядка 4 не можешь найти?
426 95610
>>95564

>Нахуя, если потом все группы порядка 4 не можешь найти?


Так это отражение сегодняшних реалий мат образования вообще. Тут на доске приводили примеры, когда люди всякие схемы и квазикогерентные пучки знают, а на простейшие вопросы по классическому алгему, ну то есть про собственно кривые, ответить не могут. То же и с гомологической алгеброй vs интуитвное геометрическое представление на основе "дедовской" комбинаторной топологии.

Когда у тебя есть всего 6-7 лет, чтобы проехаться по верхам и основам устаревшей математики и выбрать тему для пхд (и начать понимать хотя бы абстракты актуальных статей в своей области), то сидеть и ковыряться в каждой теме и её истории просто невозможно. Алюффи позволяет перепрыгнуть через частное сразу к обобщениям и мастурбации в своей китайской комнате. Особая ирония в том, что именно на это и сетовал всеми тут ненавистный Арнольд.
427 95611
>>95610

>всеми тут ненавистный


Мне казалось дядю Арнольда тут все любят.
428 95620
>>95610

> Тут на доске приводили примеры, когда люди всякие схемы и квазикогерентные пучки знают, а на простейшие вопросы по классическому алгему, ну то есть про собственно кривые, ответить не могут.


Так пучки это первокультурная математика, а алгем ваш - картофан.

> Алюффи позволяет перепрыгнуть через частное сразу к обобщениям и мастурбации в своей китайской комнате.


А почему собственно китайская комната? Есть аксиоматика, можно взять объект, который ей соответствует и доказать какие-то его свойства. По своей сути это очень условно отличается от частного примера, просто не все могут в абстрактное мышление.
429 95624
>>95610

>Тут на доске приводили примеры, когда люди всякие схемы и квазикогерентные пучки знают, а на простейшие вопросы по классическому алгему, ну то есть про собственно кривые, ответить не могут



Не напомнишь ссылку на такой пример?
А то выглядит как сказка из публицистики Арнольда
430 95637
>>95624

>Не напомнишь ссылку на такой пример?


Как минимум я приводил такие примеры в новичковом треде как из своей преподавательской практики так и из времён обсуждения с однокурсниками на семинарах. Если кинешь ссылку на его архив, я может и поковыряюсь на досуге.

>А то выглядит как сказка из публицистики Арнольда


Из этого можно сделать вывод, что у тебя собственного опыта преподавания или хотя бы обсуждения нет. Также я с этим сталкивался при чтении некоторых обсуждений вопросов на матх иксчендж и всяких ирс каналов (может, уже дохлых).
Хотя по твпему ответу уже сразу ясно, что даже если бы я тебе видео записал, ты бы ушёл в перманентный вывсёврёти.

>>95620

>просто не все могут в абстрактное мышление.


Это нелепая отговорка, а "абстрактное мышление" - придумка вроде "математического таланта". Либо ты можешь применить свои абстракции к базовым вещам, которые рассказывают школьникам на кружках, либо нет. Если нет, это значит, что у тебя неполное понимание темы. Ну и конечно же уже пошли оправдания, что у школьников просто нет гена абстрактного мышления, или что это вообще не нужно и картофан.
431 95638
>>95637
По поводу абстрактного мышления я хочу сказать, что есть правильная, арийская физика/математика, а есть подлая, еврейская физика/математика с геодезическими линиями и двадцатимерными пространствами.

Знаете ли Вы, что до поражения Германии во Второй Мировой, было два вида математики, - арийская и еврейская, - названные так по национальности своих сторонников?

Арийская Математика брала пример с естественных наук, склонялась к эмпирицизму, конечности и познаваемости мира, и работала исключительно c объектами, которые можно построить физически (например, в памяти ЭВМ или на бумаге).

Еврейская Математика же слоняется к религиозной абстракции и казуистике: всеохватывающей бесконечности, множествам, и порождаемым ими апориям. Так Еврейская Математика постулирует, что можно удвоить объект, путём перекладывания его частей, пространство делимо на "бесконечно малые", а для любого числа, Бог может создать большее число (аксиома о бесконечности).

Основатель Еврейской Математики, Гидеон Кантор, писал, что работает с "Абсолютом - непознаваемым человеком Актус Пьюриссимус, именуемым многими Богом". Примечательно, что Кантор окончил свою жизнь в психиатрической лечебнице, однако дело Кантора поддержали сионистские организации и католическая церковь, доведя до того, что сознательные германские студенты и профессора протестовали, требуя убрать еврейскую заразу из ВУЗов.

После войны, евреи сделали все возможное, чтобы уничтожить Арийскую Математику, удалив ее сторонников и подменив ее Теорией Множеств - центральной опорой Еврейской Математики. Так основатель интуиционизма, Лёйтзен Брауэр, подвергся изоляции, а результаты Русских и Английских финитистов умалчивались и не получили распространения. В русской истории от рук евреев пострадали математики Егоров (умер в гулаге), Лузин (подвергся травле и был отстранен), Флоренский (расстрелян), Есенин-Вольпин (репрессирован).

Добавлено через 9 минут
Сегодня математика стала еврейской даже по-цвету. Государства поддерживают так называемую "анти-расистскую математику", требующую например использовать еврейские имена в примерах и задачках, рассказывая при этом о великом "вкладе" еврейства в развитие математики.

Евреи, занимающиеся математикой, предпочитают всё специфическое-эльфийское. Причём презирают тех, кто занимается вещами, имеющими практическое применение. Поэтому в Советском Союзе вышел закон, по-которому в ВУЗах должно учиться евреев пропорционально их населению. Лишних отчисляли. Преподаватели евреи на мехмате в знак протеста ушли из университета и образовали НМУ (Независимый Московский Университет). Отсюда и название в мехматянском простонародье ``еврейская секта''.

Еврейские дети в СССР часто учились отдельное от детей гоев, в специальных элитных школах. Одной такой была Московская 57-й спецшкола, ученики которой не без оснований называют себя "пятидесятисемитами". Там часто преподавали выдающиеся преподаватели с мехмата.

В основании математики последнего столетия лежит знаменитая теория множеств Георга Кантора. Если вы откроете большую часть современных серьезных учебников математического и функционального анализа или топологии, или теории вероятности, то в начале почти наверняка увидите экскурс в теорию множеств. Почти вся современная математическая литература написана на теоретико-множественном языке. Камень теории множеств лежит в основании грандиозного здания современной науки.

Но в самом сердце этой самой фундаментальной вроде бы теории, лежащей в основе "царицы наук", почти сразу после ее создания были обнаружены очень серьезные парадоксы и проблемы, не преодоленные до сих пор. Уже сто лет с тех пор математика находится в состоянии перманентного кризиса, который остро воспринимается самыми выдающимися учеными. Великий немецкий математик Герман Вейль писал по этому поводу: "Мы менее чем когда-либо уверены в незыблемости наиболее глубоких оснований логики и математики. Как у всех и всего в мире, сегодня у нас есть свой кризис".

Математика говорит, что у шпекеровой последовательности есть предел? Говорит. Практика говорит, что его нет? Тоже говорит. Математика говорит, что апельсин можно удвоить путём перекладывания его частей? Говорит. Удалось кому-нибудь с новозаветных времён повторить эту процедуру? Наблюдения раз за разом показывают, что при такого рода операциях закон сохранения вещества неукоснительно соблюдается. Математика предсказывает наличие в бесконечномерном гильбертовом пространстве базиса Гамеля. Наблюдать оный пока вообще никому не удалось. Так что математика - именно лженаука, и никак иначе.

Именно уверенность в нематериальности математических объектов влечёт за собой веру в возможность "приближённых" вычислений (что чушь - вычисления бывают либо точные, либо неверные). Да, самолёты летают и при засилье Теории Множеств. Но если бы математика была конструктивной, они летали бы лучше, потому что конструкторы не забивали бы себе голову теоретико-множественным мусором, на практике бесполезным и дезориентирующим.

Аксиомы имеют смысл только тогда, когда они выражают свойства объективно существующих предметов. Так, если мы введём аксиому "на каждой руке человека содержится 3.1415… пальцев" и построим на базе этой аксиомы формальную теорию, то положения этой теории будут бессмысленны и даже вредны.
431 95638
>>95637
По поводу абстрактного мышления я хочу сказать, что есть правильная, арийская физика/математика, а есть подлая, еврейская физика/математика с геодезическими линиями и двадцатимерными пространствами.

Знаете ли Вы, что до поражения Германии во Второй Мировой, было два вида математики, - арийская и еврейская, - названные так по национальности своих сторонников?

Арийская Математика брала пример с естественных наук, склонялась к эмпирицизму, конечности и познаваемости мира, и работала исключительно c объектами, которые можно построить физически (например, в памяти ЭВМ или на бумаге).

Еврейская Математика же слоняется к религиозной абстракции и казуистике: всеохватывающей бесконечности, множествам, и порождаемым ими апориям. Так Еврейская Математика постулирует, что можно удвоить объект, путём перекладывания его частей, пространство делимо на "бесконечно малые", а для любого числа, Бог может создать большее число (аксиома о бесконечности).

Основатель Еврейской Математики, Гидеон Кантор, писал, что работает с "Абсолютом - непознаваемым человеком Актус Пьюриссимус, именуемым многими Богом". Примечательно, что Кантор окончил свою жизнь в психиатрической лечебнице, однако дело Кантора поддержали сионистские организации и католическая церковь, доведя до того, что сознательные германские студенты и профессора протестовали, требуя убрать еврейскую заразу из ВУЗов.

После войны, евреи сделали все возможное, чтобы уничтожить Арийскую Математику, удалив ее сторонников и подменив ее Теорией Множеств - центральной опорой Еврейской Математики. Так основатель интуиционизма, Лёйтзен Брауэр, подвергся изоляции, а результаты Русских и Английских финитистов умалчивались и не получили распространения. В русской истории от рук евреев пострадали математики Егоров (умер в гулаге), Лузин (подвергся травле и был отстранен), Флоренский (расстрелян), Есенин-Вольпин (репрессирован).

Добавлено через 9 минут
Сегодня математика стала еврейской даже по-цвету. Государства поддерживают так называемую "анти-расистскую математику", требующую например использовать еврейские имена в примерах и задачках, рассказывая при этом о великом "вкладе" еврейства в развитие математики.

Евреи, занимающиеся математикой, предпочитают всё специфическое-эльфийское. Причём презирают тех, кто занимается вещами, имеющими практическое применение. Поэтому в Советском Союзе вышел закон, по-которому в ВУЗах должно учиться евреев пропорционально их населению. Лишних отчисляли. Преподаватели евреи на мехмате в знак протеста ушли из университета и образовали НМУ (Независимый Московский Университет). Отсюда и название в мехматянском простонародье ``еврейская секта''.

Еврейские дети в СССР часто учились отдельное от детей гоев, в специальных элитных школах. Одной такой была Московская 57-й спецшкола, ученики которой не без оснований называют себя "пятидесятисемитами". Там часто преподавали выдающиеся преподаватели с мехмата.

В основании математики последнего столетия лежит знаменитая теория множеств Георга Кантора. Если вы откроете большую часть современных серьезных учебников математического и функционального анализа или топологии, или теории вероятности, то в начале почти наверняка увидите экскурс в теорию множеств. Почти вся современная математическая литература написана на теоретико-множественном языке. Камень теории множеств лежит в основании грандиозного здания современной науки.

Но в самом сердце этой самой фундаментальной вроде бы теории, лежащей в основе "царицы наук", почти сразу после ее создания были обнаружены очень серьезные парадоксы и проблемы, не преодоленные до сих пор. Уже сто лет с тех пор математика находится в состоянии перманентного кризиса, который остро воспринимается самыми выдающимися учеными. Великий немецкий математик Герман Вейль писал по этому поводу: "Мы менее чем когда-либо уверены в незыблемости наиболее глубоких оснований логики и математики. Как у всех и всего в мире, сегодня у нас есть свой кризис".

Математика говорит, что у шпекеровой последовательности есть предел? Говорит. Практика говорит, что его нет? Тоже говорит. Математика говорит, что апельсин можно удвоить путём перекладывания его частей? Говорит. Удалось кому-нибудь с новозаветных времён повторить эту процедуру? Наблюдения раз за разом показывают, что при такого рода операциях закон сохранения вещества неукоснительно соблюдается. Математика предсказывает наличие в бесконечномерном гильбертовом пространстве базиса Гамеля. Наблюдать оный пока вообще никому не удалось. Так что математика - именно лженаука, и никак иначе.

Именно уверенность в нематериальности математических объектов влечёт за собой веру в возможность "приближённых" вычислений (что чушь - вычисления бывают либо точные, либо неверные). Да, самолёты летают и при засилье Теории Множеств. Но если бы математика была конструктивной, они летали бы лучше, потому что конструкторы не забивали бы себе голову теоретико-множественным мусором, на практике бесполезным и дезориентирующим.

Аксиомы имеют смысл только тогда, когда они выражают свойства объективно существующих предметов. Так, если мы введём аксиому "на каждой руке человека содержится 3.1415… пальцев" и построим на базе этой аксиомы формальную теорию, то положения этой теории будут бессмысленны и даже вредны.
432 95639
>>95637

>как из своей преподавательской практики так и из времён обсуждения с однокурсниками на семинарах.



мало ли, что ты там вспоминаешь из своей практики
мы же не знаем, как было на самом деле, что ты имеешь в виду под "простейшими примерами", что ты считаешь "ответить не могут", да и было ли что-нибудь вообще. как бы твоё, анона, частное мнение относительно твоего личного (гипотетического) экспириенса это не очень убедительно, сорри. Я-то подумал, прямо тут в треде кто-то про пучки задвигал, а потом на простой ерунде посыпался, вот было бы интересно посмотреть

>Из этого можно сделать вывод, что у тебя собственного опыта преподавания или хотя бы обсуждения нет.


быстро ты выводы делаешь

>я с этим сталкивался при чтении некоторых обсуждений вопросов на матх иксчендж


тоже было бы интересно посмотреть

>Хотя по твпему ответу уже сразу ясно


я смотрю ты у нас прошаренный
433 95642
>>95639
Конкретно про группы порядка 4 писал Вербит, что один из градов в Брюсселе на этот вопрос ответить не смог никак.
>>95564-анон
434 95644
>>95642
свидетельству вербита я, конечно, доверяю полностью,
но думаю, что студент, осиливший схемы, таблицу умножения для такой группы нарисует (если его очень сильно заставят, правда, потому что это дико уныло)
15092119589261.jpg208 Кб, 799x694
435 95656
>>95642
>>95644
Вы просто недостаточно обпучкались, раз приводите такие аргументы. Абстрактная алгебра тем и хороша, что не опускается до численных примеров, а манипулирует только отношениями, и этого достаточно. Алюффи буквально это пишет на странице 52 в сноске 10 к своему примеру с группами перестановок.
436 95775
Кто из преподов в НМУ по алгебре самый прошаренный? Хочу посмотреть лекций, а хуй знает какие выбирать за последние 10 лет. У кого наиболее эстетически приятный курс?
c28.jpg113 Кб, 1280x720
437 95794
>>20 (OP)
Вопрос. Как теперь украинцу обозначать множество целых чисел???
438 96380
Пытаюсь разобраться с определением векторного пространства как модуля.
Векторное пространство над полем $F$ - это фактически абелева группа $G$ плюс гомоморфизм $\phi$ (как колец) поля $F$ в $End(G)$, кольцо эндоморфизмов $G$.
Я не понимаю, как определить, какой именно эндоморфизм соответствует какому-то элементу $F$. В общем случае, как я понял, этот гомоморфизм не сюръективен, то есть будут какие-то эндоморфзимы, которым вообще никакой элемент поля не соответствует. Также в общем случае ядро может быть нетривиально, то есть каждому эндоморфизму будет соответствовать несколько элементов поля. Верно?

Но я всё равно не могу описать для себя эндоморфизмы, не привлекая умножение на скаляр. Я же определяю умножение на скаляр как применение какого-то элемента $End(G)$. Мне всё кажется, что я использую круговое рассуждение и в итоге ссылаюсь на знакомое мне умножение на скаляр.

Вот пусть например есть векторное пространство $\mathbb{R}^2$. Тогда для $\alpha \in \mathbb{R}$ и $v \in $\mathbb{R}^2$, $\alpha v$ это применение $\phi ( \alpha) \in End(G)$ к $v$. Я про это $\phi ( \alpha)$ фактически ничего не знаю же, ну просто какой-то эндоморфизм.

Я что-то не так понимаю? Или надо дальше изучать модули, чтобы познакомиться с интересными нетривиальными примерами?
439 96422
>>95775

>прошаренный


Книжки читай, их для того и писали
440 97800
>>84669
Только что читал тред где чел рассуждал про талант и труд, а тут твой пост. Надо бы самому историю математики почитать, интересно стало. Пока по твоему посту сложилось ощущение, что математики такие же в бытовом плане люди. Часто раздолбаи, но только с большим интузиазмом и сообразительные
441 97839
>>96380

> Я не понимаю, как определить, какой именно эндоморфизм соответствует какому-то элементу $F$.



Очень просто. Берёшь элемент F, применяешь к нему гомоморфизм \phi и получаешь искому эндоморфизм.
442 97840
>>97839
*искомый
443 97841
>>95775
Так тебе прошаренный или эстетически приятный?

Хочешь почувствовать себя "умным", слушай какого-нибудь Шабата (он там без прелюдий начинает на категорном языке базарить, после чего 9/10 слушателей уёбывают в ахуе) или Городенцева (он тебе за полторы лекции весь первый семестр алгебры расскажет и ещё сверху насыпет, только ты нихуя не успеешь понять, разумеется).

А вообще выскажу возможно не очень популярное мнение, но базовые курсы в нму лучше не слушать. Они там очень... странные. Они обладают всеми теми же минусами, которыми обладают базовые курсы на математических факультетах, только в них ещё и подборка тем порой очень специфическая. Так что слушая базовые курсы только в нму ты очень сильно рискуешь не выучить вообще никакую базу. Всякую алгебру, геометрию и анализ лучше всё-таки ботать самому, параллельно слушая курс в своём вузе. Так шансы, что ты не проебёшь базу намного выше.

А в нму всё-таки лучше приходить за спец. курсами.
444 97843
>>91207

> Что почитать чтобы понять


Учебник по арифметике за 5 класс.

Хуею с местных математиков. Теперь понятно, почему вам тут всем мерещится, что программировании много математики. Для вас же два числа перемножить - уже проблема.
445 97844
>>97843
Блядь, тред перепутал. Прошу прощения. Троллинг отменяется.
446 97854
>>97841

>Шабата


>Дали концептуально правильный курс: категории, когомологии групп, точные последовательности и вообще, строжайшие определения (то есть там эпиморфизм не всегда сюрьективен, например). Умерли в итоге все, а задачи сдавали более-менее только те, кто знал алгебру уже до этого. Доходило до смешного: человек имел на матфаке автомат за алгебру, а в курсе НМУ с трудом сдавал хотя бы одну задачу из листка. Зато концептуально правильно!



А почему ты "умным" в кавычках написал? Это ты считаешь тоже плохим базовым курсом? Чего там не хватает?
447 97868
>>97854

> Это ты считаешь тоже плохим базовым курсом?


Я считаю плохой дидактической находкой накачивать непуганных перваков категорным языком с самого порога. Люди научатся умным словами вроде функтор, нормальное преобразование, копроизведение, пулбэк и т.д., но при этом абсолютно не будут понимать сути и зачем вообще всё это нужно.

Теория категорий хороша в тот момент, когда у студента уже накоплен хотя бы минимальный багаж конструкций из разных областей математики и он уже самостоятельно начинает подозревать, что конструкции эти не настолько разнородны, насколько казалось в самом начале. В идеале студент сам начинает говорить на языке диаграмм, когда ему ещё ничего не сказали про категории.

На матфаке в этом плане преподы ведут себя очень грамотно. Никакого насилования категориями не происходит, но при этом и на алгебре, и на геометрии, и уж тем более на топологии преподы мягко подводят студентов к формулировкам на категорном языке

Шабатовский же подход из разряда "группа - это однообъектный группоид, ебитесь с этим как хотите" немного неконструктивен, как по мне.
448 97885
>>97868

>Люди научатся умным словами вроде функтор, нормальное преобразование, копроизведение, пулбэк и т.д., но при этом абсолютно не будут понимать сути и зачем вообще всё это нужно.


Абсолютно с этим согласен. Дроч андерградов-математиков (а часто ещё и погромистов) на Алюффи всё это только усугубляет.
Всему своё время. Нет смысла в обобщениях, когда не знаешь примеров, которые собственно и обобщаются.
449 97903
>>97868

>при этом абсолютно не будут понимать сути и зачем вообще всё это нужно


Первые две лекции посвящены вводным сюжетам про системы полиномиальных уравнений. Предлагается поверить, что их исследование становится проще при использовании соответствующего языка.

>минимальный багаж конструкций из разных областей математики


По сути, для примеров хватает знаний конструкций из экзамена Матшкольник Вербита.

>насилования категориями


>группа - это однообъектный группоид, ебитесь с этим как хотите


Группы, кольца, модули определяются (и обсуждаются) сначала на теоретико-множественном, а потом и категорном языках.
>>97885

>Нет смысла в обобщениях, когда не знаешь примеров, которые собственно и обобщаются


Есть, дальнейшее изложение и доказательства станут проще.
Необходимые примеры в курсе есть. Не все хотят двигаться от частного к общему. Можно и нужно наоборот.
450 97910
Кто может обьяснить как находить IQR и как его искать?(тема статистика)
451 97911
Кто может обьяснить как находить IQR и как его искать?(тема статистика)
452 98668
>>43860
Уже неактуально, но может кому-то тоже интересно: ответ есть в книжке Cox Galois Theory в первых параграфах.
Правда подстановка Виета там берется из ниоткуда. До неё можно додуматься так:
Если есть общее уравнение 3 степени P(x) мы можем его попытаться упростить какой-то подстановкой x=y+k, где y новая неизвестная.
Подставив P(y+k) и разложив на множители при одинаковых степенях y увидим, что от y^2 можно избавиться, положив k=-b/3.
Получаем редуцированное уравнение y^3+py+q. Попытаемся повторить трюк, y=z+d;
:(z+d)^3=z^3+d^3+3d^2z+3z^2d=z^3+d^3+3dz(z+d)
сложив это с p(z+d) выносим (z+d)
z^3+d^3+(z+d)(3dz+p)+q
От сюда видно, что можно избавиться от (z+d) если (3dz+p)=0
Тогда d=-p/3z.
Получим z^3 - (p/3z)^3 + q

С уравнением 4 степени так уже, к сожалению, не выйдет. Обычной подстановкой можно избавиться только от кубического одночлена. Применяя подобный трюк повторно куб неизвестного будет возвращаться. Это видно, если (x+d) возвести в 4 степень, всегда будет одночлен 3 степени неизвестного.
453 98755
>>20 (OP)
Вот допустим я прочитал Винберга. А дальше что делать? (самоучка)
454 98756
>>98755
Переходи от алгебры к алгебраической топологии и геометрии.
455 98757
>>98756
Понял, спасибо.
456 98759
>>20 (OP)
Что за "Modules with algebraic K-theory in mind," которую в треде по топологии >>10844 → упоминали?
457 98762
>>98759

>Что за "Modules with algebraic K-theory in mind,"


Думаю, Berrick - An Introduction to Rings and Modules: With K-Theory in View

> Модуль можно определять до кольца


Дальше даже не читал. Самое естественное понимание модуля - это действие, т.е. гомоморфизм кольца в кольцо эндоморфизмов абелевой группы.
458 98764
>>98762

>Модуль можно определять до кольца


Вообще говоря, это будет не модуль, а абелева группа с операторами. Модуль всегда является коммутативной группой с операторами. Не менее естественно чем то, о чём ты говоришь.
459 98765
>>98762

>Думаю, Berrick - An Introduction to Rings and Modules: With K-Theory in View


Спасибо!
460 99010
.
461 99249
Где можно поподробнее прочитать про поля Галуа? Да и вообще про поля. Теорию групп я дёрнул из "Теоремы Абеля", но про теорию полей там ни слова
462 99251
>>99249
Очевидно в любой книжке о теории Галуа.
463 99272
Какая есть фундаментальная/интуитивная причина того, что внешний квадрат n-мерного векторного пространства изоморфен алгебре so(n)? Вроде как одно измеряет/представляет ориентированные площади, а другое порождает повороты.

Вроде как кажется, что интуитивно это из-за антисимметричности. С одной стороны, a∧b=-b∧a, так как меняется ориентация (или что аналогично - площадь параллелограмма, натянутого на линейно зависимые вектора, есть нуль). С другой стороны, для элементов алгебры so(n) есть косая симметрия A'=-A (где A' - транспонирование), которая есть бесконечно малый аналог определения поворотов (R'R=1).
Но это очень нестрого, и антисимметричность в этих двух случаях совершенно другая, так что это просто махание руками.

Собственно сам изоморфизм обычно приводится такой:
(a∧b)(с) = (a,c)b-(b,c)a
И это тоже мне не очень помогает. То есть это очевидно на уровне матриц, но хочется понять интуитивно почему так получается.
464 99317
>>99272
да вроде это не совсем махание руками, внешний квадрат у тебя определяется как фактор тензорной алгебры по тому соотношению что ты написал, а so(n) это типа матрицы, удовлетворяющие твоему второму условию, ну и видно, что и там и там соотношения то одинаковые: выделенная инволюция(х)=-х. Но важно понимать, что в тензорном квадрате мы факторизуем по соотношениям, а в so(n) мы не факторизуем, а выбираем из более общего объекта элементы, удовлетворяющие соотношению и изоморфизм очевидным образом получается. Более простая ситуация когда появляется такой эффект - это изоморфизм между симметрической степенью и алгеброй разделенных степеней, но там уже например если мы работаем не над полем а над кольцом, то это не верно.
Ну и да я ниче не прояснил, просто накинул говна на вентилятор.
465 99321

> махание руками


Вы из спбгу?
466 99322
>>99321
ну и че ты нам сделаешь??
467 99324
>>99322
Схожу к вам на практику
468 99392
Блять, потратил кучу времени и не смог найти Atlas of finite groups. Мне нужна полная версия со всей хуйней. Может аноны знают где достать?
469 99553
Где можно почитать про то как считать когомологии алгебры ли для конкретных алгебр (Например sl(n)) ? Мб есть методичка или еще что-то
470 99571
>>99392
А чем тебе онлайн версия не нравится?

https://brauer.maths.qmul.ac.uk/Atlas/v3/
471 99572
Как понятие "поднятие" (в алгебраической геометрии встречал это понятие и в теоркате) переводится на английский?
473 99576
>>99574
Спасибо. Я гуглил uplift, raise и всякие другие вариации, а про самый простой вариант забыл
image.png218 Кб, 1555x452
474 99899
Я правильно понимаю, что здесь неправильно перевели, и на самом деле там enriched category, т.е. обогащённая, а не оснащённая?
475 99991
>>20 (OP)
Напоминаю, что алгебра - это не математика. Вам в /sci/
476 100009
>>99991

А что тогда математика?
15122750186080.png251 Кб, 730x346
477 100053
Знаю линал там немного, немного общей алгебры. Надо что-то такое (связанное с алгеброй) прочитать, узнать, изучить за месяц, чтоб одному научнику алгебраисту рассказать-показать и он взял меня к себе. Хочу заниматься алгеброй но проебался во многих отношениях, теперь хочу нагнать и хотя бы с горем пополам что-то из себя представить туда-сюда достойное. Допучкался с вами тут, короче говоря.
Че посоветуете? Какие-то статьи, учебники, разделы в учебниках, лекции ютубовские. Хз, хоть как-то меня направьте.
478 100067
Всегда думал о действии группы как о гомоморфизме групп в симметрическую группу множества. Но мне попался следующий пост:

>(this) definition behaves poorly when one considers topological groups and continuous group actions, since in general it may not be possible to put a nice topology on \text{Aut}(Z) such that continuous actions are precisely continuous group homomorphisms G \to \text{Aut}(Z). The situation is even worse for, for example, algebraic groups and algebraic group actions.



Может кто прокомментировать?
479 100076
>>100067
Я могу объяснить на пальцах, через аналогию.

Есть в лингвистике такая штука, как теория семантических ролей. Рассмотрим в качестве примера какое-нибудь предложение с переходным глаголом, например "кузнец бьет молотом по наковальне". Его схема выглядит как "агент X действует способом Y на объект Z с помощью объекта Q". В нашем случае понятийная структура аналогична, "структура G действует на структуру Z с помощью своего элемента g способом, описанным аксиоматикой действия группы".

Действие - это обобщение понятия операции. Операция работает внутри структуры (аналогия с двумя химическими элементами, которые складываясь, превращаются в третий), а действие позволяет элементам из внешней структуры управлять поведением элементов в текущей. Способ этого внешнего воздействия описывается (или регулируется, что, в общем-то, одно и то же) некоторой аксиоматикой. Понятно, что управляющая и управляемая структуры могут быть весьма различны по уровню своей внутренней сложности - и аксиоматика, дающая некоторую "естественную" интерпретацию действия в одном случае, может не иметь ее в другом.

Группа, как абстрактный механизм, собранный из симметрий, может легко шаффлить обычное множество и вдобавок отображаться в его симметрическую группу - потому что множество, по сути, бесструктурно (если не считать "теневой структуры" попарного различия всех его элементов). Но когда ты закутываешь управляющие и управляемые структуры в некоторые дополнительные тряпки типа топологий, и при этом эти тряпки оче сильно отличаются, то начинаются всякие неожиданности. Например, далеко не факт, что две структуры, шаффлящие без разрывов третью структуру, гомеоморфны.
480 100078
>>100076
А примеры есть? И насколько часто всё идёт по пизде?
То есть допустим у нас есть действие группы G на топологическом пространстве X. Гомеоморфизмы X образуют группу Aut(X). Согласно определению выше, действием будет гомоморфизм групп G->Aut(X). Что именно здесь пойдёт не так, если на G нет топологической структуры?
481 100202
>>100067
В категории топологических пространств, насколько знаю, нет внутреннего Hom-а, то есть на множестве непрерывных отображений из произвольного топологического пространства X в произвольное топологическое пространство Y нельзя ввести структуру топологического пространства таким образом, чтобы получилась естественная биекция между Mor(X \times Y, Z) и Mor(X, Mor(Y, Z)) (Mor --- морфизмы --- это непрерывные отображения). А именно так и отождествляются два определения действия (через гомоморфизм в эндоморфизмы и через гомоморфизм из произведения). От этого «недостатка» можно избавится, ограничив класс рассматриваемых топологических пространств:
https://en.m.wikipedia.org/wiki/Compactly_generated_space
482 100900
Зачем методом Хаусхолдера для поиска собственных значений матрицы может быть нужно приведение матрицы к форме Хессенберга? Ведь в случае разложения методом исходной матрицы в вид QR на диагонали матрицы R и так будут собственные значения или нет? Вроде приведение к хессенберговой форме нужно, чтобы реализовать сдвиги для ускорения сходимости.
483 114778
Там на архиве топовые геометры начали выкладывать доказательство геометрической гипотезы Ленглэндса. Прикол, да?
484 114780
>>114778
Я не начинал

топовый геометр
image.png132 Кб, 298x438
485 114784
>>114780

>Я не начинал


>топовый геометр

486 114802
>>114778

>This paper is the second in a series of five


>439 страниц


Прикол)))
487 114807
>>114802

>топовые геометры


>This paper is the second in a series of five


>439 страниц


где-то я это уже видел только геометр был один
488 114811
>>114807
Лурье?
489 114871
Заранее извиняюсь за вопрос, но можете посоветовать, что почитать про генераторы инфинитезимальных преобразований на пространстве хороших в каком-то смысле функций на полугруппе или моноиде, типо, что известно помимо того, что это полукольцо кодифференцирований, есть ли какие-нибудь классификационные результаты?
17165359575771013.mp41,1 Мб, mp4,
512x640, 0:12
490 116034
>>20 (OP)
Есть вариант поступить в магистратуру на кафедру алгебры и топологии, скорее всего не на бюджет (да и как-то похуй). В голове держим что меня не надо отговаривать. Разделы математики сдавал на средние баллы, но уже ничего не помню, т.е. разобраться если захочу - могу. Посоветуйте, на ваш взгляд, самые подходящие прелиминарии в виде книг, видео-лекций, которые подготовят меня на моем пути. Хочу туда поступить шутки ради, оказаться в абсолютно новом пространстве ахуевше непонятных вещей, среди людей стоящих за такими дверями знаний, ключ для которых есть странное любопытство с долей аутизма.
491 116035
>>116034
там экзамены вступительные нужны? их и готовь
492 116036
>>116034
ван дер варден
493 116065
>>116034
Какие разделы математики ты сдавал? Что планируешь изучать на кафедре?
494 116066
>>116034
это ты куда поступать собрался? никогда не слышал, чтобы где-то была кафедра "алгебры и топологии"
495 116074
>>116034
Алгебра и топология:
«Algebra» - Michael Artin
«Abstract Algebra» - David S. Dummit, Richard M. Foote
«Algebra» - Serge Lang
«Topology» - James R. Munkres
«General Topology» - Stephen Willard
«Topology» - Klaus Jänich
Курсы:
MIT OpenCourseWare
Coursera и edX
496 116076
>>116074
Признаться, не вижу смысла отдельно изучать общую топологию. Не лучше в процессе изучения многообразий её пройти? Или в процессе изучения функционального анализа.
497 116090
>>116076
нет, не лучше
498 116092
>>116090
Почему не лучше?
499 116100
>>116074

>«Algebra» - Michael Artin


Хорошая книга. Хочу купить ее себе. Доставка книг с амазона в Россию сейчас работает?
500 117017
>>114778
Выглядит как чатжпт высер на основе нкатлаба. Все плевались на бурбаков, и тем не менее статьи и книги условных дьедонне, серра, или гротендика читаются комфортно и по сей день, с интересными идеями и комментариями.
А эта статья - какой-то симулякр математики.
Обновить тред
« /math/В начало тредаВеб-версияНастройки
/a//b//mu//s//vg/Все доски

Скачать тред только с превьюс превью и прикрепленными файлами

Второй вариант может долго скачиваться. Файлы будут только в живых или недавно утонувших тредах.Подробнее